Sie sind auf Seite 1von 96

Prep

for
USML
E
Forum

Resources

New Posts

Register

Login

Prep4USMLE USMLE STEP 3 Forum USMLE Step 3 Support Forum Step 3 HY Comments on Step 3
HY

Step 3 HY
0/5
1
2
3
4
5

Post Reply

Author

drtanvir

1
2
Next
Last

27 Posts
Dec 17, 2008 - 11:51 AM

#1

Forum Senior
Precocious puberty: It is very important to distinguish between the two
most common presentations and causes of precocious pubarche in order
to facilitate the proper treatment.
Precocious puberty is caused by premature activation of the
hypothalamus-pituitary-gonad (HPG) axis.
Topics: 23
Posts: 118

precocious pseudo-puberty is caused by a gonadotropin-independent


process, typically an excess of sex steroids (severe cystic acne,
significant growth acceleration). It can be caused by late-onset
congenital adrenal hyperplasia.
Hypothalamic dysfunction leading to precocious puberty is usually less
dramatic in presentation. Sequential development of the following is

typically present: testicular enlargement, penis enlargement, pubic hair


growth, and lastly, a growth spurt.

Benign premature thelarche: is characterized by bilateral breast


enlargement not accompanied by other signs of isosexual precocious
puberty. These other signs of precocious puberty include rapid increase
in height, increase in bone maturity, appearance of axillary and pubic
hair, and menstrual bleeding. The treatment for benign premature
thelarche is expectant because majority of the patients remain stable or
have reversal of the breast enlargement in a few months. Patients with
benign premature thelarche have a normal hormone profile. Their final
height is generally not compromised.
DD: 1. Hypothalamic hamartomas secrete GnRH and cause central
isosexual precocious puberty in both males and females. Central
precocious puberty is characterized by rapid acceleration of height,
increase in bone age, thelarche, adrenarche, pubarche, and menarche.
Lab investigations in patients with hypothalamic hamartoma reveal
gonadotropin levels in the pubertal range and elevated estrogen levels.
Majority of these patients will require treatment with a GnRH analog.
2. Estrogen production from an ovarian tumor can lead to peripheral
precocious puberty. This syndrome has similar clinical features as
hypothalamic hamartoma, and is characterized by accelerated height
and bone age, and menstrual bleeding. Hormonal profile reveals
elevated estradiol in the presence suppressed LH and FSH. Treatment is
usually surgical.
3. McCune-Albright syndrome consists of "caf au-lait" spots, fibrous
dysplasia of the bone, and precocious puberty. This patient does not
have any features to suggest McCune-Albright syndrome. The cause of
precocious puberty in McCune-Albright syndrome is excessive production
of estrogen from ovarian cysts.

Nocturnal enuresis can be categorized as primary or secondary, with


secondary enuresis accounting for roughly of all cases. If the child
was continent for at least 6 months prior to the onset of bed-wetting,
then the enuresis is considered secondary. While psychological problems
are very rarely the cause of primary nocturnal enuresis, they are quite
frequently the cause of secondary nocturnal enuresis. Nocturnal
enuresis is more common in boys. Prevalence declines slowly
throughout childhood; almost of 5-year-old children (such as the boy
in this case) continue to have nocturnal enuresis. Typically, the condition
resolves of its own accord between the ages of 5 and 7. By age 10,
prevalence of nocturnal enuresis has dropped to under 5% of all
children.
To increase the likelihood of success, parents should maintain a caring,
patient approach, and may wish to try behavioral modification with
positive reinforcement. If the childs nocturnal enuresis persists for some
time, alarm therapy or a prescription for desmopressin may be worth
trying.

**First line management for primary nocturnal enuresis for children less
than seven years of age is to reassure the patients parents that the
child usually outgrows this phase and spontaneously recovers. Other
options for treatment, however, are the use of alarms, along with
behavioral therapy, such as limiting the childs fluid intake before
bedtime. In the alarm method, a sensor is placed in the childs
underwear or in the bed padding. Once the child voids and moisture is
detected, the alarm is activated, waking up the child so that he could go
to the toilet before he continues to empty his bladder. Although alarms
have been shown to be less immediately effective than desmopressin
use, the former is still more effective in preventing relapses. Alarms are
more effective than treatment with tricyclics during and after treatment.

When giving an opinion about a patients diagnosis, the physician must


be cautious and try to avoid giving false reassurance or a premature
diagnosis that cannot be supported by adequate clinical evidence. In
cases wherein the patients history or physical findings do not point to a
specific diagnosis, the physician can correctly state that it is "probably a
medical condition", and then begin the work up in order to arrive at a
more specific diagnosis.

Chronic adrenal insufficiency: It must be suspected in patients with the


corresponding signs and symptoms: fatigue, weight loss, myalgias,
increased pigmentation, and a decreased amount of axillary and pubic
hair. Hyponatremia, hyperkalemia and hyperchloremic metabolic
acidosis are characteristic laboratory findings. The most common
etiology of this condition is primary adrenal insufficiency (Addisons
disease). Diagnosis can be made through the ACTH stimulation test, or
by the measurement of early morning serum cortisol level (A level less
than 10 mg/dl indicates a high probability of the disease). DD:
Hyperthyroidism shares some similar clinical manifestations with
Addisons disease (weight loss, asthenia,); however, the presence of
other findings, such as fatigue, androgen deficit signs, hyponatremia
and hyperkalemia, cannot be explained by hyperthyroidism alone.
Hypothyroidism, not hyperthyroidism, is usually related to fatigue and
hyponatremia (although the other symptoms still cannot be explained
by hypothyroidism).

Sickle cell disease: In children, the most common initial symptom of


sickle cell disease is dactylitis, which develops in 40% of patients.
Splenic sequestration is the second most common, and occurs in about
20% of patients. Other complications, such as ischemic events, are not
as frequent, and are seen mainly in the adult population.
Salmonella is an uncommon cause of hematogenous osteomyelitis in the
general population; however, it is the most common cause of
osteomyelitis in patients with sickle cell disease. Long bones are usually
affected, and multiple foci are often present. Sickle cell anemia should
be strongly suspected in child who has a history of anemia and an

episode of Salmonella osteomyelitis. Interestingly, only about 30% of


cases of sickle cell anemia are diagnosed at the age of 1 year.
Hemoglobin electrophoresis will confirm the diagnosis.

Acute otitis media: The most common complication after an episode of


acute otitis media is another episode of otitis media. Approximately 75%
of early recurrences are due to different bacteria, while other recurrent
episodes are caused by the same agents (usually Streptococcus
pneumoniae, Haemophilus influenzae, and Moraxella catarrhalis).
Children who have had more than two episodes are especially at risk.
Some authors recommend prophylactic antibiotic therapy in such cases.
Contrary to popular belief, pneumonia is not a complication of acute
otitis media. Both conditions can coexist, but one is not a risk factor for
the other.
** While adults have wide, steeply angled Eustachian tubes, infants
have shorter, almost horizontal Eustachian tubes. This fact makes fluid
accumulation and infection quite common in young children; however,
not all children develop acute otitis media (AOM). Recurrent episodes
require further exploration by the clinician.
Exposure to cigarette smoke has been shown to alter mucosa, cilia, and
adenoid structures, in addition to increasing the risk of developing AOM
and upper respiratory tract infections. An increase in the level of smoke
exposure is paralleled by an increase in the number of AOM episodes.
Exposure to cigarette smoke is one of the most important risk factors
for developing acute otitis media. Parents should be urged to quit
smoking.
Current recommendations for AOM patients with an initial clinical
treatment failure (failure of acute otitis media to respond clinically to
amoxicillin by day 3 of treatment) and who has not received antibiotics
in the month prior to the initiation of treatment are high doses of
amoxicillin/clavulanate or certain second or third generation
cephalosporins. The rationale is to enhance activity against penicillinresistant S. pneumoniae (drug resistant S. pneumoniae or DRSP) found
in an increasing percentage (30-60%) of cases of pneumococcal AOM in
the US.
Discontinue amoxicillin, begin IM ceftriaxone, and refer the boy to an
otolaryngologist for urgent tympanocentesis. This would be an
appropriate choice if the patient has an initial clinical treatment failure
and has received antibiotics in the month prior to beginning the current
treatment. (Prior antibiotics in the month preceding AOM increases the
likelihood that AOM is due to DRSP.) Under these conditions, the patient
would also have been initially treated with high dose amoxicillin, high
dose amoxicillin/clavulanate, or certain second or third generation
cephalosporins. These regimens are reported to be more effective
against DRSP than the usual dose of amoxicillin. IM ceftriaxone is even
more effective against DRSP. Moreover, immediate tympanocentesis
allows culture and sensitivity testing which can provide invaluable
guidance in selected difficult cases not responding to empiric treatment.

Tympanostomy and tubing is generally reserved for chronic otitis media


(COM) with effusion persisting for more than 3 months, or recurrent
AOM (greater than six episodes in 6 months) which is not prevented by
prophylactic antibiotics (half of normal dose amoxicillin or sulfisoxazole).
It might be considered in AOM if the TM bulging, earache, fever,
vomiting, and/or diarrhea were unusually severe or persistent.
**Post OM management: A normal appearing tympanic membrane with
decreased mobility on pneumatic otoscopy is suggestive of an effusion
in the middle ear. An effusion commonly persists up to three months
after an acute episode of otitis media (OM) has been treated. If no other
symptoms are present in such cases, watchful waiting is all that is
necessary. If other symptoms are present, the effusion is bilateral, or
has persisted for more than three months, further therapy should be
considered.
Switching to a second-line antibiotic should be considered if there is
inadequate improvement of symptoms or appearance of the tympanic
membrane, or when there is persistence of a purulent nasal discharge.
According to the CDC, three drugs can be used as alternatives. These
are amoxicillin-clavulanate, cefuroxime axetil, and intramuscular
ceftriaxone.
Tympanocentesis or myringotomy with culture are indicated in children
in whom the clinical response to a second-line treatment has been
unsatisfactory.
Hearing should be assessed if effusion has persisted for more than three
months.
Myringotomy and insertion of tympanostomy tubes should be considered
for patients with otitis media and effusion after antibiotic therapy and an
ample period of watchful waiting.

Gynecomastia is very common (up to 70%) in pubertal males. Its


occurrence during puberty is associated with the relative excess
production of estrogens from the testes. In majority of subjects,
gynecomastia regresses in about 18-24 months; however, this may
persist in some patients, and although rare, it could be cosmetically
disfiguring.
Surgical resection is required in very large, cosmetically disfiguring
gynecomastia which has not responded to medical therapy.
Liver cirrhosis is associated with gynecomastia. This is mainly due to an
increased aromatization of circulating androgens.

Methamphetamine (MF): therapy is associated with decreased weight


and height. Sadness and irritability can be seen in up to 22% of children
at therapeutic doses. MF overdose has similar effects as
dextroamphetamine, that is: anorexia, nausea, vomiting, tachycardia,
increased blood pressure, palpitations, dizziness, headache,

nervousness, euphoria, agitation, and aggressive or violent behavior.


It is very important to remember that MF cannot be stopped abruptly
because of the high prevalence of psychological dependence to the drug.
MF must therefore be slowly tapered.
DD: MF use in childhood has been associated with an increased
probability of drug abuse problems in adulthood. Illicit drug overdose,
especially cocaine, may be difficult to differentiate from MF or
amphetamine overdose. A urine toxicology test may be useful in most
cases.

Exercise-induced bronchoconstriction is usually seen after strenuous


activities or exercises. Bronchodilation initially occurs during exercise,
followed by bronchoconstriction, which begins after 3 to 5 minutes, and
rapidly peaks within 10 to 15 minutes of cessation of exercise. This is
extremely common in patients with a history of symptomatic asthma,
and the patient"s response correlates with the degree of airway
hyperresponsiveness.
The first step in the management of patients with exercise-induced
bronchoconstriction is to control the underlying asthma effectively.
Patients with well-controlled asthma and a history of exercise-induced
bronchoconstriction should have prophylactic treatment with inhaled
beta-2 agonists (e.g., albuterol) approximately 5 to 10 minutes before
the initiation of exercise. Patients with less severe or less frequent
attacks, along with their close contacts, must be taught how to
recognize and treat the acute attack with inhaled beta-2 agonists.
It is important to realize that exercise is not the cause of asthma, but it
induces bronchoconstriction in patients with underlying asthma. The
patient should not be instructed to avoid exercise. In contrast, exercise
should be encouraged to reduce the minute ventilation for any given
level of exercise, and to prevent the recurrence of exercise-induced
bronchoconstriction.

Capillary (strawberry) hemangioma: superficial, bright, red, strawberrylike round lesion is a benign vascular lesion. Most capillary
hemangiomas are evident at birth. They may undergo subsequent
growth for a period of several months (proliferation phase).
Spontaneous regression usually follows (involution phase). It is
estimated that 70% of superficial lesions will disappear by seven years
of age, although residual skin changes may be left.
For lesions that are small, do not affect normal functioning, and do not
constitute a significant cosmetic problem, observation is the best
approach.
Laser treatment should be considered in patients with lesions on the
face, and at sites of potential functional impairment (orbital area).

Imaging studies are usually indicated in infants with multiple superficial


lesions to exclude visceral involvement.
Oral corticosteroids, and sometimes subcutaneous interferon, are
indicated for the treatment of rapidly growing lesions.

Anorexia nervosa : Anorexia nervosa is a complex medical condition,


which is difficult to treat. There are some indications to admit a patient
with anorexia nervosa to the hospital. The Society for Adolescent
Medicine (SAM) has developed some guidelines. The indications for
admission are: dehydration, electrolyte abnormalities (hypokalemia,
hyponatremia), bradycardia (less than 50 beats per minute),
hypotension (BP < 80/50 mmHg), hypothermia (under 96F),
orthostatism, acute food refusal, severe malnutrition (weight < 75% of
the average for her age, height and sex), acute medical or psychiatric
emergencies (pancreatitis, seizures, syncope, suicidal ideation,
psychosis), and cardiac arrhythmia. If the patient meets any of these
criteria, she must be admitted immediately. All other medical and
psychiatric evaluations may be done after the patient is admitted to the
hospital.
Hypocalcemia is common in patients with anorexia nervosa because of
the presence of hypoalbuminemia.
Refeeding syndrome can develop during the second or third week of
nutritional therapy of patients with anorexia nervosa. It is characterized
by edema and heart failure, and can lead to delirium and cardiac arrest.
Phosphate must be replaced immediately, and the patient should be
managed in the intensive care unit.

Neonatal sepsis: The diagnosis of neonatal sepsis is suspected when an


infant has poor appetite, decreased reactivity to external stimuli,
diminished oral intake, and lassitude or depressed sensorium. The most
common etiologic agents are Group B Streptococcus, Escherichia coli,
Klebsiella and Enterobacter species. Although Listeria infections are rare
in the United States, ampicillin is recommended as part of the antibiotic
regimen. Cefotaxime, ceftriaxone or antipseudomonal penicillins are
used in combination with the aforementioned drug. The type of
bilirubinemia involved also affects the approach in management of
septic infants. Indirect hyperbilirubinemia is usually physiologic in nature
and persists, while direct hyperbilirubinemia is probably induced by
sepsis-related cholestasis.
Ceftriaxone should not be used if there is hyperbilirubinemia, because it
will increase both types of bilirubin. Moxalactam and sulfonamides are
not recommended due to their capability of increasing indirect bilirubin
levels.

Sydenhams chorea (SC; also known as Saint Vitus dance), one of the
classic manifestations of rheumatic fever. SC is more common in girls,

especially between 5 and 13 years of age. SC is usually preceded by a


period of emotional lability, manifested by a tendency to cry or laugh, as
well as deterioration of school performance. SC starts with distal hand
movements, later progressing to facial jerking and grimacing, as well as
abnormal feet movement. Chorea usually develops several (one to
eight) months after the onset of acute rheumatic fever, whereas carditis
and arthritis typically develop within 21 days. The presence of carditis is
diagnostic. Approximately one third of the patients have carditis; mitral
regurgitation is the most common cardiac finding. Patients may not
remember a prior sore throat, and cultures are also usually negative.
Patients usually have hypotonia, and the relaxation phase of the patellar
reflex is usually delayed. When the patient is examined with the arms
stretched out, the arms move from the prone to the supine position
(positive pronator sign).
DD: 1. Gilles de la Tourette syndrome starts between 2 and 15 years of
age, and is four times more frequent in boys. Pronator sign, carditis and
hypotonia are not present.
2. Attention deficit disorder is a chronic condition with no abnormal
findings on physical examination.
Sydenhams chorea (SC) should be treated immediately with oral
penicillin for ten days. If patient adherence is an issue, an alternative is
a single dose of intramuscular Benzathine penicillin. In patients who are
allergic to penicillin, weight based erythromycin, divided in 2-4 doses,
should be given. SC usually resolves within 12 to 15 weeks; however,
prophylaxis should be continued into adulthood.
If the motor function is severely compromised, valproic acid,
phenobarbital or haloperidol can be started, and the patient must be
referred to a neurologist.

Atopic dermatitis: The characteristic (red, oozing) rash may involve the
face and the scalp in children. Scaly, dry cheeks, especially in the winter,
are also typical. Atopic dermatitis is believed to have a strong
allergic/immunologic component because many patients demonstrate
allergies to food and inhalant allergens. 80-85% of patients have
elevated IgE levels, and there is a strong family predisposition to the
disease. The condition promptly responds to topical steroids and
calcineurin inhibitors such as topical tacrolimus and pimecrolimus.

Diarrhea diet: In a child with diarrhea and no clinical signs of


dehydration, preferred intake includes fluids low in sugar and the
resumption of a normal, age-appropriate diet with plenty of complex
carbohydrates with limited fats and sugars. Foods containing excessive
sugar can increase the osmolarity of the stool in the intestinal lumen,
and fatty foods are known to delay gastric emptying. Both can worsen
diarrhea.
The previously popular BRAT diet is no longer recommended for
refeeding children with diarrhea because it is quite low in calories and

protein. Moreover, the bananas and applesauce in the BRAT diet add
excessive sugar.
Of greater concern is loperamides linkage with paralytic ileus, toxic
megacolon, CNS depression, coma, and death in children.

Rabie prophylaxis: In a case of exposure to bats, immediate vaccination


is recommended if the person was not aware of the presence of the
animal. Postexposure prophylaxis consists of the administration of one
dose of anti-rabies immunoglobulin and five doses of anti-rabies
vaccine. If treatment is postponed for no more than five days (if the
patient did not immediately report the exposure), effectiveness will be
the same, although there have been reports of rabies in persons not
vaccinated in the first 72 hours when the bite was in the head or the
neck.

Ethics: Permission to treat a child can only be granted by the parent or


legal guardian. As an exception to this rule, adolescents are typically
allowed to give consent for their own care in regards to pregnancy,
contraception, sexually transmitted diseases, substance use, and
emotional illness.
When providing medical care, clinicians must seek to balance the
autonomy of the family with the welfare of the child. In true emergency
situations requiring imminent treatment of a child, most medical teams
choose to proceed with treatment despite the parental refusal of care.
Very rarely do courts uphold charges against physicians in such cases.

Homosexual parents: Homosexuality was classified as a sexual disorder


in the DSM I and II, but was removed from the revised DSM III.
Multiple studies of both biological and adopted children of homosexual
parents have been performed, and the general consensus is that these
children experience normal childhood development. Among children of
homosexual parents, adolescent concerns about identity and peer
approval are becoming less intense as variance in the American family
unit grows more common and socially accepted.
The majority of children adopted by homosexuals consider themselves
to be heterosexual, though some do describe themselves as homosexual
or bisexual.
Homosexual parents have not been proven to be more passive than
heterosexual parents, and there is no documentation of increased
violence or antisocial behavior in their children. Studies of domestic
violence suggest it is similarly prevalent in homosexual and
heterosexual relationships.
Although at least one study has suggested that certain learning
disabilities (e.g., dyslexia, stuttering) are more common in homosexuals

than in heterosexuals, there is no evidence that this tendency translates


to increased learning disabilities in the children of homosexual parents.

Bacterial conjunctivitis: Characterized by redness and copious, purulent


discharge. No fever or blurred vision. Examination shows conjunctival
erythema and yellow exudates. It is usually a self-limited disease;
however, it must be treated because there is a small, but real risk of
keratitis, which can lead to visual impairment.
Erythromycin ointment or sulfa drops are the first line of therapy for
uncomplicated bacterial conjunctivitis, as they cover most organisms.
Fluoroquinolone eye drops are preferred for contact lens wearers and
cornel ulcers because of the activity against pseudomonas. They should
not be used in uncomplicated bacterial conjunctivitis as first line therapy
because of the risk of the emergence of resistance.
Primary care physicians should not prescribe corticosteroids, as they can
cause sight-threatening complications in patients with bacterial
conjunctivitis and herpes keratitis.
Bacterial conjunctivitis is very contagious. The best thing to do is to
keep the child at home until the discharge is cleared. However, in many
patients who work outside, this may not be feasible. In those patients,
at least 24-hours of topical antibiotic therapy should be applied before
returning to work.

Delayed puberty: The earliest sign of puberty is nocturnal increase in


the LH surge. This is followed by a daytime increase in the levels of
gonadotrophins, and increase in testosterone levels. On physical
examination, enlargement of testes is one of the earliest signs of onset
of puberty.
A patient is diagnosed with delayed puberty if he does not have
testicular enlargement by 14 years of age, or if his testicles are 2.5 cm
or less in diameter. Another criterion is a delay in the development for 5
years or more from the onset of genitalia enlargement. The most
common cause of delayed puberty is constitutional delay. The initial
evaluation involves the use of an imaging test to determine bone age.
Bone age that is older or equal to the chronological age warrants further
testing to rule out chromosomal abnormality and endocrine causes.
DD: Constitutional pubertal delay is characterized by delayed puberty,
retarded bone age, and a positive family history without any evidence of
systemic disorder. These patients develop puberty without any
intervention. The development of secondary sexual characteristics can
be enhanced in patients with constitutional pubertal delay by using small
doses of testosterone for a short time. The use of testosterone for a
short period of time does not compromise the final adult height.
Kallmans syndrome consists of anosmia with hypogonadotropic
hypogonadism, and sometimes there are associated midline facial

defects. This syndrome is due to a genetic defect which leads to the


defective migration of GnRH-secreting and olfactory neurons to their
final adult positions.

Neurofibromatosis: To meet the diagnostic criteria NF 1, two or more of


the following are required: a first degree relative with NF 1, the
development of more than six CALS of 5 mm in greatest diameter (if
children) or 15 mm (if adult), or the presence of more than two
neurofibromas, Lisch nodules, optic glioma, bone dysplasia, or axillary
freckling. If the patient is a child, there is a 15% possibility of
development of optic nerve gliomas. Current recommendations include
having a detailed ophthalmologic evaluation for early detection of this
problem and for search of Lisch nodules (iris hamartoma). As primary
care physicians have a lack of expertise in such an evaluation,
immediate ophthalmologic referral is advised.
If there are any ophthalmologic or neurologic abnormalities, other tests
such as MRI, head CT scan, EMG, or neurosurgery evaluation may be
needed.
DD: Hypopigmented spots, in combination with a family history of
bilateral deafness, strongly suggest neurofibromatosis type 2 (NF-2), an
autosomal-dominant disorder. The spots described actually represent
caf-au-lait spots that are usually hypopigmented (unlike the
hyperpigmented caf-au-lait spots found in NF-1). Deafness is caused
by bilateral acoustic neuromas, a characteristic neurologic manifestation
of the syndrome.

Herpes zoster or varicella: The diagnosis of herpes zoster or varicella is


made clinically. No further laboratory tests are needed.
In immunosuppressed patients (e.g., HIV), the rash can be atypical and
can be confused with herpes simplex. If the patient is critical and
antiviral therapy needs to be started, lesions can be scraped for PCR or
immunofluorescence studies.

Vaccination: There are two true contraindications for DTaP vaccination:


1) anaphylaxis within seven days of administration of a previous DTP or
DTaP vaccine, and 2) encephalopathy within seven days of
administration of a previous DTP or DTaP vaccine. If high fever
(>104.8F), shock, inconsolable crying for three or more hours, or
seizure occur within 24 hours of receiving a dose of DTaP vaccine, then
subsequent doses should be given with caution. Temporary
contraindications to receipt of the DTaP vaccine include moderate or
severe illness, with the vaccine administered as soon as the illness
resolves. Mild acute illness with or without fever is not a contraindication
to vaccine administration. There are no significant contraindications to
receipt of vaccinations against Hib, poliovirus, and pneumococcus;
therefore, a four-month-old child with otitis media should be vaccinated
against DTaP, Hib, IPV, and PCV.

**Varicella vaccination of household contacts of transplant recipients is


relatively safe and is currently recommended by the American
Association of Pediatrics (AAP). It is advisable for all patients to receive
pre-transplantation immunization, although this is not always possible
and durable protection is reached in less than 50% of patients.
Transmission of vaccine-associated VZV is not typical, except for some
cases when post-vaccination rash appears; therefore, this child should
be monitored for the appearance of a rash and isolated if a rash appears

TB:
All patients with sputum-positive pulmonary or laryngeal tuberculosis
can transmit the infection to other household contacts or healthcare
workers via infectious aerosols containing Mycobacterium tuberculosis
bacilli. The acts of coughing, sneezing, singing, and even speaking can
all produce microscopic aerosols containing the organism. All such
patients should be placed in respiratory isolation until they are
confirmed to be non-infectious. Patients are rendered non-infectious if
they are receiving effective antituberculous therapy and have had three
consecutive negative results on sputum acid-fast smears performed on
different occasions. Non-infectivity should be documented by serial
negative acid-fast smears, and not by the duration of therapy. A patient
can still be infectious after prolonged drug therapy if he has a drugresistant infection. Resolution of chest x- ray findings in patients with
pulmonary tuberculosis typically lag behind the clinical response. Chest
radiograph changes can persist for longer periods of time after the
patient has been rendered non-infectious, and may even become
permanent.
**Multi-drug resistant TB: is an emerging problem in the USA. If the
organisms are resistant to both rifampin and INH, we call it a multi-drug
resistant TB (MDR-TB). In case of exposure to MDR-TB,
chemoprophylaxis with pyrazinamide and ethambutol or quinolone with
anti-mycobacterial activity, like ofloxacin or levofloxacin is
recommended. But, if it is resistant to INH only, then the standard
recommendations are to give the chemoprophylaxis with Rifampicin
alone for four months or Rifampicin plus PZA for two months. The
problem with the rifampin plus PZA combination in an
immunocompetent host is liver toxicity. The patient needs careful
monitoring.
**Tuberculous meningitis: is one of the forms of extrapulmonary
tuberculosis. It is associated with high morbidity and mortality rates.
Patients usually present with an insidious onset of symptoms such as
malaise, headache, and low-grade fever. If left untreated, these
symptoms can rapidly progress to persistent headaches, vomiting,
cranial nerve involvement, confusion, seizures, coma, and eventually,
death within six to eight weeks of the onset of illness. All patients with
suspected tuberculous meningitis (based on initial history and CSF
examination) should be immediately started on empiric antituberculous
therapy, pending the results of confirmatory tests. The prognosis of the
patient with tuberculous meningitis greatly depends on the stage in
which the treatment is initiated. Early diagnosis and treatment of the

patients is critical to prevent adverse clinical outcomes.


All patients with tuberculous meningitis should be initially treated with a
combination of isoniazid, rifampin, and pyrazinamide (bactericidal
agents) for the first two months, followed by treatment with INH and
rifampin for the succeeding months. These three agents have good CSF
penetration, and can achieve adequate therapeutic levels in the
cerebrospinal fluid. Based on the current guidelines, all infants and
children with drug-sensitive tuberculous meningitis should be treated for
at least 12 months. In a patient with drug-resistant infection, the
therapy may be extended to 18-24 months, depending on the clinical
response and severity of the illness.
Summary: 1. Specific antituberculous therapy should be initiated early
in all patients with clinically suspected tuberculous meningitis.
2. Infants and children with tuberculous meningitis, miliary TB, and
tuberculous osteomyelitis should receive 12 months of antituberculous
therapy.

kyphosis:
a) Flexible:
Scenario: Physical examination shows a postural round back that is
corrected by voluntary hyperextension. Forward bending reveals no
lateral deformity and no angulation. Neurological examination of the
lower extremities is normal. You order x-ray of the spine that shows a
convex alignment of the thoracic spine at 35 degrees.
It is typically noticed by parents or teachers who observe adolescents
sitting or standing in a slouched position. On lateral radiographs, the
angle of thoracic kyphosis is normal or slightly increased (normal 20-40
degree). Interestingly, contrary to common belief, there is no evidence
that flexible kyphosis leads to adverse physical effects or permanent
deformity.
b) Scheuermann disease (structural kyphosis )
Scenario: Physical examination reveals thoracic curving of the spine that
is not corrected with voluntary hyperextension of the spine. On forward
bending, sharp angularity is observed in the thoracic region, but no
lateral deformity is present. Neurological examination of the lower
extremities is normal. X-ray of the spine shows a convex alignment of
the thoracic spine at 55 degrees.
Unlike flexible kyphosis, structural kyFphosis is not corrected with
voluntary efforts, and a sharp angulation is commonly seen on forward
bending. The typical treatment for structural kyphosis that is not severe
(less than 70-80 degrees) includes the use of a Milwaukee brace. In
more severe cases (significant angulation, intractable pain, neurological
abnormalities), surgical correction is used.

Turner syndrome (45,XO): dorsal feet and hands edema, short webbed
neck, and a cardiac murmur) is typical for Turner syndrome. It
represents monosomy for the X chromosome (45, X), that is why no
Barr body is revealed on the buccal smear. Less common chromosomal
abnormalities that can be present in patients with Turner syndrome
include X chromosome mosaicism and Xp deletion. Interestingly, the risk
of having an infant with monosomy for the X chromosome does not
increase with advance maternal age, unlike Downs syndrome and
Klinefelters syndrome (47, XXY). Moreover, no increased recurrence risk
is present after having an infant with Turner syndrome.
All patients who are initially diagnosed must be screened for the
presence of other associated somatic abnormalities. The most important
of these are the cardiac defects, which include coarctation of the aorta,
bicuspid aortic valve, mitral valve prolapse, and hypoplastic heart. Since
some of these defects cannot be picked up by clinical examination, an
echocardiogram is necessary. The other associated defects are: visual
and hearing deficits, kidney malformation (including horseshoe shaped
kidney), and an increased predisposition for autoimmune
endocrinopathy (especially primary hypothyroidism); therefore, in
addition to an echocardiogram, all patients initially diagnosed with
Turner syndrome require visual and hearing assessment, renal
ultrasound, and TSH level measurement.
Patients with Turner syndrome develop moderate to severe insulin
resistance and diabetes when they are older. Blood sugar screening is
performed based on the patient"s clinical manifestations.
Patients with Turner syndrome have hypogonadism, and eventually
require estrogen replacement; however, if estrogen therapy is given at
an early age, there may be premature fusion of the epiphysis, which will
potentially decrease the patient"s final height. Most physicians begin
prescribing hormone replacement therapy when the patient reaches 14
years of age. Growth hormone is approved for use in patients with
Turners syndrome to improve their final height.

Imaging studies in UTI:


Imaging studies are recommended in the following patients:
1. children under the age of five years with a febrile UTI
2. males of any age with a first UTI
3. females under the age of three years with a first UTI
4. children with UTI who do not promptly respond to antibiotic therapy
5. children with recurrent UTI
Voiding cystourethrogram is routinely recommended, and is considered
by most authors as an important imaging study to demonstrate vesicoureteral reflux (VUR).

Infantile hypertrophic pyloric stenosis (IHPS): usually presents in infants


who are three to six weeks old. 83% of affected patients are boys.
Postprandial, non-bilious vomiting is characteristic. Peristaltic waves can
sometimes be detected just before vomiting. These occur as wavy
movements traveling from left to right across the upper abdomen. A
palpable "olive-like" mass in the right upper quadrant is pathognomonic
of the disease, but is actually detected by only 49% of physicians, since
most cases are diagnosed at an early stage (usually between the third
and fourth week).
Historically, laboratory evaluation will show hypokalemia and a
hypochloremic, metabolic alkalosis secondary to the loss of gastric
hydrochloric acid, although electrolyte imbalances are seen less often
now that the diagnosis is made earlier. The imaging modality used most
commonly to diagnose IHPS is ultrasound, though an upper
gastrointestinal (UGI) contrast study is preferred in some medical
centers.
Electrolyte derangements and dehydration must be corrected before
proceeding with surgical correction of infantile hypertrophic pyloric
stenosis. Studies have shown that children who undergo surgery without
first correcting the electrolyte imbalances are at increased risk for
postoperative apnea.
Rx: pyloromyotomy
Endoscopically guided balloon dilation has been studied, but is not often
used because of inconsistent success in opening the muscular ring of the
pylorus. This approach is typically reserved for patients who are not
otherwise good surgical candidates.
Studies have documented an association between the development of
infantile hypertrophic pyloric stenosis and the usage of oral
erythromycin, which is typically given as postexposure prophylaxis for
pertussis. In addition, there is some indication that the usage of
macrolides in breastfeeding women is linked to the development of
infantile hypertrophic pyloric stenosis, especially in infant girls.

Constipation: The normal frequency of passage of stools in the infant is


around six to eight times daily. This approximates to one stool passage
per episode of breastfeeding. On the fourth week of life, the pattern
changes. The stool frequency decreases to one or two episodes daily or
even less, such as three episodes per week. This has to be differentiated
from true constipation in order to avoid exaggerated concern.
DD: Hirschsprungs disease usually presents earlier in life (failure to
pass meconium in first 48 hours) as intestinal obstruction or
enterocolitis. Notwithstanding, some patients will be diagnosed later, but
will have a history of chronic constipation and failure to thrive. For this
reason, observation is required to determine if the constipation persists.
**While constipation can signify a serious disorder (e.g., Hirschsprungs
disease, cystic fibrosis, hypothyroidism) in select circumstances, most

children who are constipated are normal. Children with a history of


resisting toilet training are at increased risk for developing constipation,
as are those children whose dietary intake is high in dairy products and
low in fiber. If dietary modification fails to relieve constipation, usage of
a laxative is the next best step. Once the childs bowel begins to operate
properly on its own, the laxatives should be discontinued gradually.
Magnesium hydroxide, also known as milk of magnesia, is a mild saline
laxative that causes the osmotic retention of fluid in the gut lumen. It
can be titrated to produce soft, but non-liquid stool, and is safe for use
in young children. In a child experiencing moderate constipation and no
encopresis, magnesium hydroxide is an excellent treatment choice.
Bisacodyl suppositories are powerful stimulant laxatives that cause
increased peristalsis. Usage of these suppositories commonly results in
abdominal cramping, diarrhea, and nausea, and is best reserved for
short-term "rescue therapy" in children with severe constipation.
Phosphate enemas are highly effective at relieving impaction, and act by
distending the rectum and stimulating the muscles of the colon;
however, frequent use of these enemas can cause electrolyte
imbalances, and is therefore discouraged. This form of treatment is best
reserved for relieving impaction. or for bowel preparation prior to
medical procedures.

Tick bite: The risk of acquiring Lyme disease after being bitten by a tick
is less than 1.5%. The most common complication of tick bites is local
inflammation or infection.
In order to be infected with Lyme disease, the patient must have been
exposed to the tick for more than 36 hours, because the transmission of
the infectious agent - Borrelia burgdorferi - takes place only after the
tick is firmly attached to the skin and has suctioned a certain amount of
blood which gives it an engorged appearance. If a tick is found and it is
not engorged with blood, there is no risk of Lyme disease, as ticks take
at least 24 hours to firmly attach to their victims.
The majority of patients have a transitory skin reaction in the first 24-72
hours on the site of contact with the tick; this lesion must not be
confounded with erythema chronicus migrans, which develops later.
The tick that transmits Lyme disease is brown, while the one that
transmits RMSF is black

Epiglottitis: With the introduction of the Haemophilus influenzae type b


vaccine in 1985, epiglottitis has become much less common in recent
years. When it does occur, it causes an inflammatory edema of the
epiglottis that impinges upon the airway. Respiratory arrest is easily
provoked at this stage, and treatment should be focused on relieving
any airway obstruction and treating the infection. The threshold for
performing intubation should be very low. Since the outpatient setting is
inadequate for such procedures, it is imperative that an ambulance be

called so the childs condition can be properly addressed by emergency


personnel.
Frequently, epiglottitis is diagnosed solely by the clinical presentation.
However, if a lateral neck radiograph is deemed necessary, it will reveal
a swollen epiglottis (the "thumbprint sign") in classic epiglottitis.
Endotracheal intubation should be performed before the radiograph is
obtained, because patient is at great risk for respiratory arrest.

Lead intoxication: Lead intoxication remains a common preventable


environmental hazard, although its prevalence is declining in developed
countries. Early recognition of lead poisoning helps to prevent the
development of long-term irreversible neurological sequelae. The two
main factors contributing to lead poisoning in industrialized countries
are exposure to old, lead-based paints which are usually found in old
houses, and non-nutritive hand-to-mouth activities of young children.
Children with a blood lead level greater than 10 g/dL should be
comprehensively evaluated by obtaining a CBC, serum electrolyte levels,
and urinalysis. Environmental/behavioral interventions are crucial in the
management of patients with lead intoxication. Ideally, these include
identifying the source of exposure, removing the family from the leadcontaining dwelling, cleansing of the environment, and educating the
family in an effort to reduce hand-to-mouth activitiesof the children.
High-risk populations must be screened for lead intoxication. This
includes children who live in or frequently visit old houses, and children
whose sibling was diagnosed with lead poisoning. Measurement of the
blood lead level is the gold standard in lead intoxication screening. A
blood lead level greater than 10 g/dL is diagnostic. Venous sampling is
always preferred to capillary sampling because the former is more
accurate.
**Mild lead poisoning (blood lead level < 45 g/dL) can be treated by
DMSA or d-penicillamine, although this approach is not evidence-based.
Moderate lead poisoning (blood lead level between 45 and 70 g/dL)
requires the treatment with EDTA IV or oral DMSA. Continuous IV
infusion of EDTA is preferred to IM injection
Severe lead intoxication (blood lead level > 70 g/dL and/or acute
encephalopathy) is a medical emergency and prompt chelation therapy
with dimercaprol and EDTA is necessary.

Afebrile pneumonia syndrome (APS): Causative organisms of APS


include Chlamydia trachomatis, Ureaplasma urealyticum, Mycoplasma,
CMV, RSV, Parainfluenza virus, Adenovirus, and Pneumocystis carinii.
The fever may either be absent or low grade. Other symptoms include
cough, tachypnea, irritability, and poor feeding. Because of the lack of
specificity of symptoms and radiologic findings, the causative organism
cannot be determined solely by the clinical presentation. However, some
organisms have characteristic features which may aid in the differential
diagnosis and direct the necessary work-up of the patient.

The age of onset (between 2 and 19 weeks) and the presence of


staccato cough are suggestive of Chlamydia pneumonia. This diagnosis
can further be supported if there is a history of conjunctivitis in the
neonatal period. Conjunctivitis is present in half of cases and may
present either in the neonatal period or be concurrent with the
pulmonary infection. Additional characteristics include auscultatory and
radiologic findings that are out of proportion to the overall healthy
appearance of the infant. Chest-x ray shows hyperinflation, peribronchial
thickening, and bilateral, symmetrical, interstitial infiltrates. The WBC
count is usually normal, but the eosinophil count is elevated. The
diagnosis can be confirmed by tissue culture isolation of the organism
from nasopharyngeal specimens, direct fluorescent antibody tests,
enzyme-linked immunoassays, or polymerase chain reaction.

Delayed speech development: may indicate an underlying hearing


impairment and should prompt referral of the patient for audiologic
assessment. Other candidates for hearing evaluation include children
with a family history of hereditary childhood hearing loss, history of
meningitis, history of recurrent or persistent otitis media with effusion
for more than three months, documented intrauterine infections,
craniofacial anomalies, and use of ototoxic medications such as
aminoglycoside.
Hearing loss early in life, even mild or unilateral, may affect speech and
language development, social and emotional development, and
academic achievements. Early identification is thus mandatory for a
better prognosis; hence, screening programs have been widely and
strongly advocated. Although the American Academy of Pediatrics
endorses universal screening, at present, systematic screening is
implemented in 32 states only.
The etiology of hearing impairment depends on whether the hearing loss
is conductive or sensorineural. The most common cause of conductive
hearing loss in children is presence of fluid in the middle ear. Other
causes include tympanic membrane perforation (trauma or infection),
ossicular discontinuity (infection, cholesteatoma, and trauma),
tympanosclerosis, and congenital anomalies of the external ear canal or
middle ear components. As for sensorineural hearing loss (SNHL), it
may be congenital or acquired. The most common infectious cause of
congenital SNHL is CMV. Other less common congenital infectious causes
include toxoplasmosis, syphilis, and rubella. Postnatal infectious causes
include group B streptococcus and bacterial meningitis. Hearing loss can
also be genetically determined; it may be autosomal dominant or
recessive, isolated or in a syndromic association with other anomalies.

Downs syndrome: Endocardial cushion defect is the most common


congenital heart lesion in patients with Downs syndrome. Other
malformations that are frequent in Downs syndrome include duodenal
atresia, Hirschsprungs disease, atlanto-axial instability, and
hypothyroidism. Echocardiography is the most useful test for diagnostic
evaluation of this condition. Cardiac catheterization is evidently more

accurate than echocardiography for diagnostic evaluation, but this


modality is reserved for cases in which the size of the shunt is
uncertain, laboratory data and clinical findings are equivocal, or when
pulmonary vascular disease is suspected.
Patients with Downs syndrome are at increased risk of developing acute
leukemia later in life. Other conditions which occur with a higher
frequency in Down"s syndrome include Alzheimer-like dementia, autism,
ADHD, depressive disorder, and seizure disorder.

Infectious mononucleosis: Suspect IM in any adolescent who presents


with fever, pharyngitis, tonsillitis, lymphadenopathy, splenomegaly, and
rash. The recognition of "atypical lymphocytes" in the peripheral smear
can be a clue to its identification, but these can also be seen in
toxoplasmosis, CMV infection, or lymphocytic leukemia.
**It is infrequent to find posterior cervical adenopathy in a case of
common bacterial pharyngitis. The development of rash after
administration of amoxicillin is also one of the characteristics of the
disease. Even though the spleen and liver are not always palpable, the
former is constantly increased in size and friable. Since there is an
increased risk of splenic rupture, especially if the patient is actively
involved in sports, a patient who is highly suspicious for IM must be
recommended to avoid practicing sports while waiting for the test
results. Despite this fact, almost 50% of the cases of splenic rupture
associated with IM can occur spontaneously, without evidence of
previous trauma or excessive exercise.

Smoking cessation: Nicotine is a highly addictive drug. It is extremely


difficult for even the highly motivated to stop smoking. Numerous
factors may elicit the urge to smoke. Studies have shown that smokers
who attempt to quit while working or living with people who continue to
smoke are much more likely to relapse.
Smoking cessation is difficult to accomplish if the smoker remains
subject to environmental triggers, including friends and family who
continue to smoke.
Parents who want their children to stop smoking should quit
themselves.
Prescriptions for nicotine replacement (e.g., nicotine patch, nicotine
gum) should be offered to those smokers interested in quitting.

Rash: Due to the success of immunization campaigns, infection with the


rubella virus (Choice B) is now very rarely seen in the United States;
cases today typically arise in non-immunized foreign born populations.
Infection with rubella is most often asymptomatic, but can cause
German measles, a mild illness classically characterized by low-grade
fever, lymphadenopathy involving the posterior cervical and occipital

lymph nodes, and a maculopapular rash that begins on the face and
spreads caudally. The rash is similar in appearance to that caused by
measles, but patients are usually much less sick upon presentation with
rubella. Supportive treatment is sufficient when the illness is selflimited. Of primary concern is the congenital rubella syndrome, which
has devastating effects on the unborn child, especially when a nonimmune pregnant woman is exposed to the virus in the first trimester.
Varicella, commonly known as chickenpox, is an illness characterized by
a low-grade fever, malaise, and a macular rash that appears in crops
which progress through several stages, including papules, vesicles,
pustules, and crusts. The rash is considered distinctive because it
includes a variety of lesions at different stages.
Rubeola, commonly known as measles, is an illness characterized by the
three Cs: cough, conjunctivitis, and coryza. Fever and photophobia are
also common. The blue-white Koplik spots found on the buccal mucosa
precede the appearance of the maculopapular rash, which starts on the
face and spreads caudally to the trunk and extremities.
Roseola is a mild illness characterized by a high fever that rapidly
resolves. The fever is followed by the eruption of a characteristic rosy
nonpruritic rash originating on the trunk and spreading to involve the
extremities.
Rocky Mountain Spotted Fever is a rickettsial disease transmitted by tick
bite. It is characterized by fever, myalgias, headache, and a petechial
rash. Classically, the rash first involves the distal extremities (especially
the palms and soles) and subsequently spreads to involve the trunk.

Henoch-Schnlein purpura Classical clinical manifestations of HenochSchnlein purpura include abdominal pain, arthralgias, skin lesions, and
renal involvement. An antecedent upper respiratory infection is present
in 50% of patients. Abdominal pain is a presenting symptom in 10-15%
of patients. The skin lesions are symmetric, involve dependent parts of
the body, and classically progress from an erythematous, macular rash
to papular purpura. The joints and kidneys are also commonly involved.
Thrombotic thrombocytopenic purpura (TTP) is a serious disorder
characterized by the following classical pentad:
1. Severe thrombocytopenia
2. Microangiopathic hemolytic anemia (RBC fragments)
3. Fluctuating neurological signs
4. Renal failure
5. Fever
Patients with TTP generally present with fever, pallor, petechiae, and
confusion. The peripheral smear shows RBC fragments. PT and PTT are
usually normal. The LDH is elevated due to hemolysis. Hemolytic uremic

syndrome (HUS) and TTP comes under a spectrum of diseases. If a


patient has more neurologic symptoms and less renal failure, the
disease is considered TTP. On the other hand, if a patient has significant
renal failure and less neurologic symptoms, the disease is considered
HUS. Both TTP and HUS are very serious conditions, and require
emergent plasmapheresis.

Idiopathic thrombocytopenic purpura Platelet-specific autoantibodies are


the presumed pathogenesis of idiopathic thrombocytopenic purpura. In
children, the condition is typically characterized by a sudden onset of
bleeding, manifested as petechiae, purpura, epistaxis, and gingival
bleeding. More severe bleeding is rare. Commonly, there is a history of
infection in the several weeks prior to presentation. Symptomatic
patients with moderate to severe thrombocytopenia

drtanvir

Dec 17, 2008 - 11:53 AM

Forum Senior

Diaper rash (diaper dermatitis): is a type of irritant contact


dermatitis that is caused by a combination of factors:
overhydration, friction, maceration and prolonged contact
with excretions. The appearance of the rash after an episode
of diarrhea is very characteristic. Management includes
keeping the diaper area of the skin as dry as possible. This
involves: frequent changing of diapers, avoiding tight-fitting
diapers, exposing the skin to air, using diapers with super
absorbent surfaces, and applying barrier creams such as zinc
oxide or petrolatum. An uncomplicated diaper rash resolves
quickly if the abovementioned measures are performed.
Otherwise, a low-potency corticosteroid ointment may be
prescribed.
DD: Candidal diaper rash: Tomato-red plaques and satellite
papules are characteristic of candidal intertrigo and perineal
infection. This infection is common in infants who have
recently received antibiotic therapy because the decrease in
the normal bacterial flora favors yeast proliferation. The
condition can be confounded with diaper dermatitis, which
affects the same area but usually spares the crural folds.
Treatment of candidal diaper rash involves the application of
an antimycotic cream such as clotrimazole or nystatin, while
zinc oxide is the preferred treatment for diaper dermatitis.

Topics: 23
Posts: 118

#2

Intestinal parasitosis: due to Helminths (hookworms,


roundworms, pinworms, or whipworms) is the most frequent
cause of chronic diarrhea, iron-deficiency anemia, and
eosinophilia in patients coming from endemic countries. The
prognosis is excellent with adequate therapy. Since the
disease is easily transmitted, prophylactic treatment should
be given to the rest of the household members.
Hemophilia: is an X-linked recessive disorder that occurs
almost exclusively in males. Females are usually the carriers
of the disease. A female may acquire hemophilia, an
extremely rare possibility, only when her father is a
hemophiliac, and her mother is a carrier who transmits the
abnormal allele to her.
When a father is a hemophiliac, he will transmit his abnormal
X gene to all his daughters, and consequently, they will all be
carriers of the disease. When the mother is a carrier, her
daughters have 50% chances of being carriers and her sons
have 50% chances of getting the disease.
Childhood absence epilepsy (CAE): age of onset (4-8 years),
no neurologic signs, typical EEG pattern, and no myoclonic
activity. This condition is usually responsive to ethosuximide
or valproate, although high doses may be required to control
it effectively. Interestingly, many traditional anti-epileptic
drugs are not effective in absence epilepsy, and can even
exacerbate the condition (e.g. gabapentin). The prognosis in
patients with CAE, especially if generalized tonic-clonic
seizures are absent, is good. Staring spells will disappear in
the teenage years. The risk of persistence of the condition is
higher in patients who develop generalized tonic-clonic
seizures. Unlike CAE, juvenile myoclonic epilepsy (JME) is
characterized by a late onset of absence seizures with
myoclonic activity, and is associated with life-long seizures.
Candidiasis: The first-line therapy for oral candidiasis is a
topical antifungal - nystatin suspension or clotrimazole
troches. Oral fluconazole can be used in resistant cases.
Child abuse: Always maintain a high index of suspicion for
physical/sexual abuse in children (especially females) with

sudden behavioral problems, especially if the family has an


unstable economic background or if the child"s parents have a
history of drug/alcohol abuse.
**Whenever you suspect child abuse, it is strongly
recommended that you inform the social services of your
hospital or to call the child protection agency. Even if you are
proved wrong later on, you do not owe any liability legally
and, perhaps, it is the most important step in the
management of suspected child abuse.
Allergic rhinitis: Dog or cat dander is one of the most common
identifiable allergen in patients with allergic rhinitis. Allergen
avoidance is traditionally considered the first step in the
management. If the allergen is not identified or the symptoms
persist after the avoidance measures, nasal corticosteroids
should be the first-line therapy. Second-generation
antihistamines and cromolyn are less effective than topical
steroids. Nasal decongestive sprays are not recommended
because tachyphylaxis usually develops and rebound
phenomena may result.
Thalassemia major: Patients with this disorder usually die
early in life due to relentless anemia and catastrophic
expansion of erythroid precursors. Hypertransfusion therapy
has a great role in treated patients (i.e. the child may survive
several years after the diagnosis). Hypertransfusion regimen
can suppress the effects of chronic severe anemia and
extramedullary hematopoiesis, but result in significant iron
overload and resultant organ damage.
Failure to thrive (FTT): is not a diagnosis in itself; rather, it is
a term used to describe failure to gain weight in children
younger than two years old. Children categorized as FTT
weigh less than the 5th percentile for their age; more severe
cases involve a slowing of linear growth and head
circumference as well. The three causes of FTT are inadequate
calorie intake, inadequate calorie absorption, and increased
calorie requirements. Newborn infants need 110 kcal/kg/day,
while children up to twelve months need 100 kcal/kg/day to
grow at a normal pace. Psychosocial factors are very
commonly involved in cases of FTT; this is why the clinician
must explore whether there are stressors in the home

environment. Organic causes of FTT, while less common, may


include feeding problems, milk-protein intolerance, inborn
errors of metabolism, infection, cystic fibrosis,
gastroesophageal reflux, or renal tubular acidosis.
Dietary modification is the first-line of treatment in an
otherwise well-appearing child with FTT.
The anxiety caused to the family by admitting the child to the
hospital should not be underestimated. Although admitting a
child with FTT for hyperalimentation was a common practice
in the past, it is now considered unnecessary unless the child
is severely malnourished, or if the child is at risk for neglect
or abuse. If oral feedings do not result in appropriate weight
gain within four to six weeks, supplemental feeding by
nasogastric tube is preferred to the intravenous route.
If the history and physical examination are suggestive of an
organic cause for the childs FTT, then laboratory
evaluation is warranted. Appropriate tests include urinalysis
and culture, hematocrit, blood urea nitrogen, calcium,
electrolyte levels, HIV ELISA test, and Mantoux tuberculin
skin testing. Note that unless there is clinical evidence
suggestive of thyroid disturbance, thyroid hormone levels are
not typically tested.
Cat bites: get infected most of the time. Furthermore, the
lesion is usually deeper than when produced by a dog,
because the cats teeth are smaller and sharper.
Pasteurella multocida, the major causative organism, is
transmitted by the cats bite. However, this is often a
polymicrobial infection. Adequate prophylaxis can be obtained
by using amoxicillin and clavulanic acid. In adults, especially
those allergic to penicillin, the second alternative is
doxycycline. More ill patients can be treated with intravenous
ampicillin-sulbactam combination.
** The incidence of infection complicating a cat bite is close to
50%; therefore, antibiotic prophylaxis is usually
recommended, especially in high-risk situations (a hand bite
is considered a high-risk situation) wherein the potential
infection could extend to the bone and joint. The first dose of
antibiotics (e.g. ampicillin/sulbactam) is usually given
parenterally, and covers a broad spectrum of cat mouth flora,
including Pasteurella multocida. An oral antibiotic (e.g.
amoxicillin/clavulanate) is administered subsequently for 3-5

days.
Cat-scratch disease: is an infection that usually affects the
young immunocompetent population. It is produced by
Bartonella henselae. Around 10% of the patients with catscratch disease can develop suppuration of the lymph nodes.
Other complications are: visual loss due to neuroretinitis,
encephalopathy, fever of unknown origin, and
hepatosplenomegaly.
Gastroesophageal reflux (GER) is a clinical diagnosis.
Reassurance should be offered to the mother that the
"spitting up" is a normal occurrence in infants up to 24
months old. It typically requires no intervention if the child is
otherwise healthy and developing appropriately (the "happy
spitter"). Children with mild GER symptoms, should be initially
addressed with reassurance and thickening of formula with
cereal, which usually results in decreased emesis, decreased
cry, and better weight gain.
Prone positioning is another conservative treatment that may
alleviate symptoms; however, this treatment method is of
some concern, because of the correlation between prone
positioning and SIDS. Formula thickening should be
attempted first.
H2 receptor antagonists such as ranitidine are appropriate in
those infants with a more severe GER presentation and who
have failed conservative treatment.
Surgery is reserved for cases of GER that do not respond to
medical management.
Pneumonia: It can be difficult to distinguish between bacterial
and viral pneumonia, and indeed viral pneumonia is very
common in previously healthy children and adults.
Classically, bacterial pneumonias are sudden in onset,
associated with high fevers, and cause the child to look very
ill if not toxic. Auscultatory findings are typically focal and
distinctive. Chest radiographs may demonstrate a lobar
consolidation. In contrast, viral pneumonias are gradual in
onset and cause the child to look mildly ill. Auscultatory
findings are more diffuse and bilateral. Chest radiographs may
also demonstrate a more diffuse, bilateral infiltrate. Both
forms of pneumonia are frequently preceded by an upper

respiratory tract infection.


Community-acquired bacterial pneumonia in children is most
commonly caused by S. pneumoniae. Amoxicillin is the drug
of choice for the outpatient treatment of patients younger
than five years of age.
Group B streptococcus pneumonia occurs most often in
neonates.
Mycoplasma causes respiratory infection most often in schoolaged children and young adults. The clinical presentation is
usually gradual (although it may be abrupt) and is
characterized by malaise, headache, fever, rhinorrhea, and
sore throat with progression to lower respiratory symptoms.
Pneumocystis carinii pneumonia (PCP) is seen in patients with
defects in cell-mediated immunity, especially in HIV-infected
patients. It is usually seen in HIV patients when the CD4 cell
count decreases to less than 200 cells/cubic mm. It has an
insidious onset, and usually presents with low-grade fever,
cough, dyspnea, and tachypnea. Chest x-ray results of
patients with PCP reveal diffuse, bilateral ground-glass
opacity.
Trimethoprim-sulfamethoxazole is the initial drug of choice for
treatment in patients with suspected or proven Pneumocystis
carinii pneumonia.
Respiratory syncytial virus is the most important respiratory
pathogen of early childhood, and the major cause of
bronchiolitis and pneumonia in children < 1 year. The illness
manifests in annual outbreaks, appearing in winter with peaks
in January, February, or March. Although this disease affects
all age groups, lower respiratory symptoms appear mainly in
children. Apnea is an important clue for this condition, as up
to 25% of infants presenting with RSV infection will have this.
How it causes significant apnea is not clearly known. The
illness tends to be more severe, with a higher frequency of
complications in preterms and infants with chronic medical
conditions such as congenital heart disease, chronic lung
disease, and immune deficiencies.
Diagnosis of RSV is quickly made by detection of RSV antigen
in nasal or pulmonary secretions by ELISA.
**In healthy infants and young children, bronchiolitis is
usually a self-limited disease; however, patients who are
hypoxic or cannot feed because of distress should be
hospitalized. These children should be kept in respiratory

isolation. Therapy in most cases consists of supportive


measures. Humidified oxygen and tube or intravenous
feedings are indicated. Although there is no strong evidence
that inhaled bronchodilators are effective in patients with
bronchiolitis, it is a routine practice to administer these
(nebulized albuterol or epinephrine) and observe the patient
for any effect. If no prompt clinical response is seen, most
clinicians discontinue these drugs.
Although ribavirin is a nucleoside analogue with good in vitro
activity against RSV, studies examining its effect in children
have been conflicting, and the cost for a course of therapy is
substantial. It is usually reserved for patients with severe
disease.
A secondary bacterial infection of the middle ear and future
risk of bronchial hyperreactivity are the most common
complications of RSV bronchiolitis.
Congenital toxoplasmosis: The consumption of undercooked
meat during pregnancy may be associated with congenital
toxoplasmosis, which can manifest as microcephaly or other
abnormalities, such as chorioretinitis, mental retardation,
deafness, and seizures. Domestic cats are definite hosts for T.
gondii. Humans can acquire the infection by: (1) the
consumption of raw or undercooked meat of infected animals
(including lamb, beef, or game), or (2) contact with cat feces.
Other congenital infections of the TORCH group can also
result in microcephaly.
Rehumatic disease prophylaxis: The benefits of antibiotic
therapy in a child with acute pharyngitis include: (1)
reduction of the severity and duration of the symptoms, (2)
prevention of rheumatic fever, and (3) prevention of local
suppurative complications.
Interestingly, antibiotic therapy does not decrease the risk of
acute glomerulonephritis because cutaneous (not pharyngeal)
infection is typically implicated as its cause.
Disease associations: Meconium ileus is usually the earliest
manifestation of cystic fibrosis (CF), and is almost
pathognomonic for the disease. Uncomplicated meconium
ileus is characterized by distal intestinal obstruction, wherein
the terminal ileum is dilated and filled with thick, tar-like,

inspissated meconium. Plain abdominal x-ray findings


(dilated, gas-filled loops of small bowel, absent air-fluid
levels, and a meconium mass within the right side of the
abdomen) are usually suggestive of the diagnosis. Since CF is
inherited in an autosomal recessive pattern, a family history
of recurrent respiratory infections (or other manifestations of
CF) is an important clue to the diagnosis of this patient.
Duodenal atresia is associated with Downs syndrome and
polyhydramnios.
A family history of severe constipation is sometimes present
in patients with Hirschsprungs disease.
Patients with pyloric stenosis are typically firstborn males.
Even though most cases of muscular dystrophy such as
Duchennes, Beckers, or myotonic dystrophy present
around 10-12 years of age, some cases can be diagnosed
when the child is 2-3 years old. Limb-girdle or
facioscapulohumeral muscular dystrophies can be detected at
this age; while congenital dystrophies and glycogen-storage
diseases are evident soon after birth, and have early
mortality.
Muscular dystrophy can be a cause of inability to walk or
limping in the infant or toddler, even though the more
common cause of this problem is spasticity, a condition
amenable to rehabilitation therapy. Hypothyroidism, MSUP
and phenylketonuria must be screened for at birth, and are
almost always accompanied by mental retardation and global
delays in development.
Neonatal screening for hypothyroidism is performed by
obtaining a small sample (few drops) of blood from the heel
pad and using a piece of filter paper to absorb the blood
sample. This test may be done within two to five days
following delivery. It is performed after the first 24 hours of
life because there is a normal physiologic TSH surge following
delivery. After 24 hours, the TSH levels gradually drop to
normal levels or may remain slightly elevated for the next few
days. It is very important to avoid any delays in the diagnosis
and treatment to avoid permanent neurological deficits.
Most centers in the United States use total T4 measurement
as a primary tool for screening for neonatal hypothyroidism. If
the total T4 levels are low, TSH levels of the same sample is

measured, and if the TSH levels are over 20 U/L, repeat


testing (measurement of both Free T4 and TSH levels) is
performed from a regular blood draw to confirm the
diagnosis. Some programs use TSH as a primary screening
tool.
Once the diagnosis of neonatal hypothyroidism is confirmed
by measuring the Free T4 and TSH levels from a regular blood
draw, other ancillary procedures such as thyroid
ultrasonography, radioactive iodine scanning and urinary
iodide levels are performed to investigate the etiology of the
hypothyroid state. Radioactive iodine uptake and scanning are
useful to determine the location, size and function of the
thyroid gland.
Levothyroxine is the treatment of choice for hypothyroidism,
and the dose in the neonate is higher than the adult dose;
however, levothyroxine therapy should be started once the
diagnosis is confirmed with a regular blood draw. The clinical
features of hypothyroidism in neonates can be subtle and
unreliable. Levothyroxine therapy in neonates is therefore
started once the biochemical confirmation is obtained, even if
the clinical features of hypothyroidism are not present.
Febrile seizures
The generally accepted criteria for febrile seizures are:
1. Age less than six years
2. No past history of afebrile seizures
3. Temperature greater than 38 C
4. No evidence of CNS infection / inflammation
5. No metabolic disturbances present which may produce
seizures
Febrile seizures may be subdivided into 2 forms: simple and
complex.
A simple febrile seizure is characterized by the absence of
focal features, a duration of less than 15 minutes for an
isolated event, and for seizures occurring in series, a
cumulative duration of less than 30 minutes. This form of
seizures is more common, and is associated with only a mild
elevation of the risk for subsequent epilepsy (i.e. afebrile
seizures) in latter life, compared to the general
population,contrary to previous thinking.
It is important to note that the question specifically asks
about prognosis, and not recurrence. If the question

particularly asks about the recurrence of febrile seizures


during childhood, then the answer would be significantly
elevated risk, regardless if this case was a simple or
complex febrile seizure for the first time.
Febrile seizures often occur on the first day of a mild illness as
the childs temperature is rising, and may be the first
manifestation of illness. Generally, studies are not necessary
for the evaluation of the simple febrile seizure, although it
may be appropriate to explore the cause of the fever in
children with other symptoms (e.g., severe diarrhea). Parents
should be reassured that it is not an indication of future
neurologic dysfunction or disease.
Complex febrile seizures, on the other hand, are characterized
by focal features (i.e. postictal paresis), a duration of more
than 15 minutes, and if occurring in series, a cumulative
duration of greater than 30 minutes.
Congenital adrenal hyperplasia (CAH) is a group of disorders
characterized by a deficiency in one of the enzymes involved
in steroid synthesis. In 90% of cases, the deficiency concerns
21-hydroxylase, which is necessary in mineralocorticoid and
glucocorticoid synthesis. In addition to a low production of
mineralocorticoids and glucocorticoids, the deficiency of 21hydroxylase results in an excess of its substrates. These
excess substrates are shunted towards androgen synthesis,
resulting in ambiguous genitalia and virilization in females,
and precocious puberty in males. ACTH levels are also
evidently elevated, and lead to hyperplasia of the adrenal
glands, further contributing to the increased production of
androgens.
Salt wasting syndrome occurs in severe deficiencies. It
presents in the first 2-4 weeks of life with emesis,
dehydration, and shock. Laboratory work-up reveals
hyponatremia and hyperkalemia (from lack of aldosterone),
and hypoglycemia (from lack of cortisol).
Diagnosis confirmation of 21-hydroxylase deficiency is carried
out by documenting the elevation of 17 alphahydroxyprogesterone, a substrate of 21-hydroxylase. Other
biologic disturbances occuring in 21-hydroxylase deficiency
include elevated corticotropin and renin levels, and a serum
aldosterone level that is inappropriately low for the renin
level.

The treatment principle for 21-hydroxylase deficiency is based


on supplementation of deficient hormones (mineralocorticoids
and glucocorticoids). This supplementation will decrease
corticotropin (ACTH) production, and consequently correct
androgen levels.
11-hydroxylase deficiency is a far less frequent cause of CAH
than 21-hydroxylase deficiency. It is characterized by: (1)
decreased aldosterone and cortisol, and (2) increased
androgens and deoxycorticosterone. A deficiency in 11hydroxylase results in an excess in its substrate, 11deoxycorticosterone. This substrate has mineralocorticoid
activity. Patients with this condition present with
hypertension, hypernatremia, and hypokalemia.
3-beta-hydroxysteroid dehydrogenase is a rare cause of CAH.
It involves: (1) a decrease in testosterone, mineralocorticoids
and glucocorticoids, and (2) an increase in DHEA-S.
Consequently, males fail to acquire normal external genitalia
because of a lack of testosterone, and females are slightly
masculinized because of DHEA-S excess.
Marfan syndrome (MFS): a disease characterized by
arachnodactyly, increased arm span relative to height, and
valvular (mitral or aortic) insufficiency. Dural ectasia is the
most common finding, although frequently overlooked
(present in more than 90% of patients), and usually requires
an MRI of the lumbar spine for the confirmation of the
diagnosis. Other characteristics that can affect up to 80% of
the patients with MFS are ectopia lentis, aortic dilatation
(more frequent in adults), and mitral valve prolapse.
The risk of aortic dissection is high in patients with MFS. For
this reason, corrective surgery is recommended when the
aortic root reaches 45 mm. About 80% of the patients will
have mitral insufficiency, which can lead to CHF. These
patients will benefit from mitral valve replacement.
Diaphragmatic paralysis in a newborn usually results from
phrenic nerve injury. The two most common causes of phrenic
nerve injury are birth injury and cardiothoracic injury . It is
typically accompanied by the signs of brachial plexus injury,
like Erbs palsy. Other causes of diaphragmatic paralysis,
(e.g., diaphragmatic hypoplasia, neural and neuromuscular
disorders) are very rare.

Myelomeningocele: In 80% of myelomeningocele cases, the


lumbar region is involved. Almost all these patients will have
bladder dysfunction, which can ultimately lead to upper
urinary tract involvement and renal dysfunction. Children with
S2-S3 involvement can have external anal sphincter
dysfunction that can lead to fecal incontinence. This is a much
less common complication than bladder involvement, and is
specifically associated with S2-S3 involvement.
Giardiasis has several routes of transmission: person-toperson, food-borne and waterborne. Person-to-person
transmission occurs in two settings: (1) in institutions where
there is fecal incontinence and poor hygiene (e.g., some
daycare centers), and (2) in male homosexuals. Symptomatic
patients with giardiasis should receive appropriate treatment.
Asymptomatic carriers of Giardia lamblia are not usually
treated, except in specific instances such as in outbreak
control and for prevention of household transmission by
toddlers to pregnant women and patients with
hypogammaglobulinemia or cystic fibrosis.
Severe malnutrition: remains as one of the major health
problems in developing countries.
The initial treatment of patients with severe malnutrition
should address the following issues: temperature control
(warming), possible infection, dehydration and malnutrition
(feeding). Dehydration should be treated with oral rehydration
whenever possible.
Posterior urethral valves are predominantly found in males
and are the most common cause of severe obstructive
uropathy in children. The abnormal development of the valves
in utero can obstruct urinary flow, leading to detrusor
hypertrophy and, eventually, vesicoureteral reflux and
hydronephrosis. Hallmarks of PUV include a distended bladder
and a weak urine stream.
Voiding cystourethrogram (VCUG) is a radiographic
examination of the bladder and lower urinary tract. The
bladder is filled with contrast material by catheter and
multiple radiographic images of the bladder and urethra are
obtained as the patient empties the bladder. VCUG is

especially helpful in evaluating young children for


vesicoureteral reflux and posterior urethral valves.
Foreign body aspiration:
Never forget your ABCs of resuscitation no matter what the
nature of emergency is.
The Heimlich maneuver is recommended in kids older than
one year. Below that age, give blows on the back with chest
thrusts.
If the child is unconscious, do it while the patient is lying
down. After the abdominal thrusts, examine the airway for a
foreign body. If visualized, it should be removed.
Transient synovitis (also known as toxic synovitis) is a
common condition that causes pain in the hip, thigh, or knee
in boys aged 3-10 years old. Up to 25% of children with
transient synovitis will have bilateral effusions. The disorder
may be preceded by a respiratory infection, although the
erythrocyte sedimentation rate (ESR) and white blood count
(WBC) are typically normal. If the clinical presentation
suggests transient synovitis and plain radiographs are
unremarkable, an ultrasound should be performed.
Ultrasonography is the preferred technique for identifying
small joint effusions, and may demonstrate widening of the
joint space of the hip. This modality is also useful in guiding
aspiration of the joint, a procedure that is warranted in a
febrile child with hip effusion.
Breastfeeding: Every primary care physician should
encourage breastfeeding. It is recommended that the infant
be fed at least every four hours. The mother should be able to
identify early signs of hunger, like suckling of the hand or
fingers or arm movements towards the mouth, as preterm or
debilitated infants may not be able to cry vigorously or show
agitation.
It is a good practice to feed the newborn using both breasts,
as complete emptying of the breast will increase the
milks nutritional qualities. Unfortunately, sometimes
there may be a decreased production of milk, and the baby
has to be fed using artificial formula. Feeding the baby using
both breasts will therefore not guarantee the infants
satiety.

Neonatal jaundice appearing in the first 24 hours of birth is


always pathologic. Immune or non-immune hemolysis is
frequently present in such patients.
G-6-PD deficiency is the most common red cell enzymopathy
that can lead to hemolysis. It is an X-linked disorder, and
should be suspected in a male infant of African,
Mediterranean, or Asian descent. No triggering agent is
usually present, although those infants who develop severe
jaundice with G6PD deficiency usually have Gilbert"s
syndrome as well.
Physiologic jaundice manifests 24 hours after birth.
Enterovirus and Arbovirus infections are the most common
cause of viral meningitis or encephalitis in the pediatric
population. These infections are more frequent during
summer, late spring, and early fall. Most arbovirus infections
are zoonosis (transmitted through animal vectors); for this
reason, these infections are more common in the rural areas.
Herpes simplex virus is the most common cause of viral
meningitis in the adult population, not in children.
Sickle cell disease is an inherited disorder that results from
the presence of hemoglobin S. The most severe and common
form is sickle cell anemia, in which only hemoglobin S is
present (HbSS). Clinical manifestations rarely present prior to
6 months of age, when fetal hemoglobin levels decline and
functional asplenia typically develops. Dactylitis (pain in the
hands and feet) is common in the first 18 months of life. The
best diagnostic test is hemoglobin electrophoresis because it
precisely determines which form of sickle cell disease is
present.
Acute severe anemia may be superimposed on chronic
anemia in patients with sickle cell disease. When this occurs,
it is a life-threatening condition. Clinical manifestations
include weakness, pallor and lethargy. The three typical
causes of acute anemia are splenic sequestration crisis,
aplastic crisis, and hyperhemolytic crisis.
Aplastic crisis represents a transient failure of erythropoiesis
with an abrupt reduction in the blood hemoglobin and the
number of erythroid precursors in the bone marrow. A very
important finding during a crisis is the virtual absence of

reticulocytes. The typical cause is an infection; parvovirus B19


infection is the most common cause in children.
Intussusception occurs when a part of the small bowel is
telescoped into itself. The most frequent vicinity is the
ileocecal region. Almost 60% of the cases occur in children
younger than 1 year of age, while 80% present before 2 years
of age. The classic presentation is a sudden onset of colicky
abdominal pain followed by vomiting, and the vomitus rapidly
changes from alimentary to bilious. The child usually draws up
his legs towards the abdomen because of the pain.
Hematochezia is present in 70% of the cases; 15% of the
patients have the characteristic "currant jelly" stools. Initially,
the child is irritable, but he may eventually become apathetic
when he is no longer able to eat, and he gets dehydrated.
Palpation of a sausage-shaped abdominal mass on the right
side is characteristic. Abdominal x-ray reveals generalized
distension of the bowel loops or another evidence of
obstruction. An enhanced density that projects into the air
level of the large bowel can be identified in 25% of the cases.
This density represents the invaginated bowel loop, and is
known as the "crescent sign."
DD: Acute gastroenteritis (AGE) presents with fever that
precedes abdominal pain and diarrhea. AGE will not explain
the presence of the abdominal mass and the massive bowel
distention seen in the abdominal x-ray of this patient.
Meckels diverticulum usually presents in toddlers and
older children, generally between five and ten years of age,
and in young adults. The most common location of pain is in
the right lower quadrant. Hematochezia is also present, but
the stool is non-mucoid, and there is no palpable abdominal
mass.
Erbs palsy: The classical scenario of Erbs palsy is
described. It is the most common form of obstetrical brachial
plexus injury and involves the upper roots (C5, C6, and
sometimes C7) of the plexus resulting in an adducted and
internally rotated right arm with forearm pronation and flexed
wrist. A serious complication is diaphragmatic paralysis due to
phrenic nerve involvement. Symmetric palmar grasp reflex
indicates that lower roots of the brachial plexus are intact and
is a good prognostic sign. The prognosis of Erbs palsy is

typically good with an 80% chance of full or near-full


recovery. Horners syndrome may be associated with the
injury of lower roots of the brachial plexus.
Gastrointestinal foreign bodies occur typically in toddlers,
because exploring objects by putting them into the mouth is a
characteristic behavior pattern in that age group. Coins are
the most common GI foreign bodies. Foreign bodies can
become lodged in any of the areas of esophageal physiologic
narrowing. They require attention and frequently removal in
that case. Up to 90% of foreign bodies that have made it into
the stomach will be passed without difficulty. Pyloric
obstruction is rare and manifests as persistent vomiting.
Up to 90% of foreign bodies that have made it into the
stomach will be passed without difficulty; therefore, no
intervention is usually necessary.
Cellulitis: is characterized by its irregular and elevated
borders, as well as increased local temperature, tenderness,
and erythema. It is an infection located between the
subcutaneous tissue and the fat, therefore local anesthesia
will not be useful in this situation. Pain can be treated by oral
analgesics.
Cleft lip with or without cleft palate is typically a multifactorial
disorder but can have autosomal dominant, autosomal
recessive, and X-linked inheritance, as well as be associated
with teratogenic agents. Reconstruction of a cleft lip is
generally performed at approximately three months of age
according to the rule of 10: 10 lbs of weight, 10
weeks of age, and 10 g of hemoglobin.
The reticulocyte count is an index of RBC production by the
bone marrow. Increased destruction of RBCs (hemolytic
anemia) prompts the normal bone marrow to compensate for
the loss by increasing the rate of erythropoiesis, which results
in an increased reticulocyte count. On the other hand,
nonhemolytic anemias, due to a decreased production of
RBCs, will result in a low reticulocyte count. The reticulocyte
count is therefore increased in hemolytic anemia, and
decreased in nonhemolytic anemia.

Diabetic ketoacidosis (DKA) must be suspected in any child


with dehydration, metabolic acidosis, nausea, vomiting,
tachypnea without oxygenation problems, abdominal pain,
and hyperglycemia. Patients must be immediately admitted to
the Intensive Care Unit for correction of the metabolic
acidosis and dehydration. Treatment involves intravenous
fluids and insulin administration, as well as adequate
potassium supplementation (to avoid hypokalemia).
Botulism: Giving honey to the child is the main clue to the
diagnosis. Constipation is typically the first manifestation of
the disease, and is followed by lethargy, poor sucking and
weak crying. Gag reflex is frequently impaired. This may
result in aspiration if airways are not protected.
Chlamydial infection This newborn most likely has conjunctival
and pulmonary chlamydial infection. Chlamydia may have
been transmitted from the mother during the neonate"s
passage through the birth canal. Cervical carriage in the
mother has been associated with a higher incidence of
preterm labor, PROM, and late post-term endometritis.
Chlamydia is the most common causative agent of infectious
neonatal conjunctivitis. Infected infants have a 25-50% risk of
developing conjunctivitis and a 5-20% risk of developing
pneumonia. Chlamydial conjunctivitis develops a few days to
several weeks after birth and manifests with conjunctival
congestion, edema, and mucoid or frank purulent discharge.
Pneumonia usually appears 3-19 weeks after birth, and
symptoms include cough, tachypnea, rales, and absence of
fever. Wheezing is rare. The latter two features differentiate
chlamydial pneumonia from RSV bronchiolitis.
Neonatal disease is best prevented by early detection and
treatment in pregnant women. Screening is recommended in
high-risk individuals such as single mothers, adolescents,
patients with low socioeconomic status, and promiscuous
patients. Erythromycin is the drug of choice in pregnancy and
should be given to both the patient and her sexual partner.
Another test should be performed later in pregnancy, and a
positive result warrants a second course of antibiotics. Infants
should also be treated with oral erythromycin for 14 days, as
topical agents are ineffective. Colonization takes place during
delivery, when the newborn comes in contact with the vaginal

secretions of the mother.

drtanvir

Dec 17, 2008 - 12:18 PM

Forum Senior

This concludes peds.

#3

Topics: 23
Posts: 118

abusedpostdo
c
Forum Senior

Dec 17, 2008 - 4:49 PM

#4

Thank you for your detailed post. What is the main source of
the above posted material?

Topics: 12
Posts: 121

drtanvir

Dec 18, 2008 - 2:15 AM

Forum Senior

WC abusedpostdoc
these are taken from MCQs Step3.

#5

Topics: 23
Posts: 118

abusedpostdo
c
Forum Senior

Dec 18, 2008 - 3:57 AM

Thank you very much! You are awesome.


What does "WC" stand for?

#6

Topics: 12
Posts: 121

drtanvir

Dec 18, 2008 - 12:16 PM

Forum Senior

Here comes the OBG


Preconception: All women who wish to get pregnant should
take 400 micrograms of folic acid daily at least one month
before the projected date of conception. Oral contraceptive
pills (OCPs) can be stopped at any time; however, conception
may be delayed because the patient may continue to have
anovulatory cycles immediately after discontinuation.

Topics: 23
Posts: 118

#7

Screening: Alpha-Feto-Protein (AFP) is produced by the yolk


sac and fetal liver and a certain amount of it crosses the
placenta to the maternal circulation. AFP production is altered
in the presence of fetal anomalies, and its measurement in
maternal serum can be used to screen for these anomalies.
Increased levels are seen in the presence of neural tube
defects, abdominal wall defects (Gastroschisis, Omphalocele),
as well as false positive causes such as fetal demise,
multiple gestation, and inaccurate gestational age. In the case
of an increase in MSAFP levels, the physician should first
perform an ultrasonography to rule out the false positive
causes, and to detect the presence of any anomaly that may
be seen by ultrasound. Amniocentesis has to be ordered
afterwards for confirmation by measuring amniotic levels of
AFP and Acetylcholinesterase (AChE). AChE is a protein that is
increased only in neural tube defects.
Low levels of MSAFP are frequently associated with
chromosomal anomalies, especially Downs syndrome.
The screening can be rendered more accurate when MSAFP is
coupled with beta-hCG and Unconjugated Estriol (UE3) levels;
it is then called triple test. A combination of a decreased
MSAFP, increased beta-hCG and decreased UE3 levels is
typical of Downs syndrome, whereas in trisomy 18, all
three parameters are decreased. Likewise, Ultrasonography
has to be performed to rule out inaccurate dates and fetal

demise, then amniocentesis to confirm the diagnosis. MSAFP


and triple test should be performed between the 16th and
18th week of gestation, and it must be noted that their
reliability in screening depends greatly on the accuracy of the
gestational age.
Amniocentesis is indicated in cases of abnormal levels of
MSAFP or triple screen, but after ultrasonography has
ascertained gestational age accuracy, and ruled out fetal
demise and multiple gestation. It is best performed between
16 and 20 weeks gestation.
Chorionic villous sampling (CVS): Chorionic villus sampling is
indicated in women with known genetic diseases, or
previously affected children. It is performed between 10 and
12 weeks gestation, and therefore offers the advantage
of a first trimester termination of pregnancy in case the fetus
is affected.Transverse limb abnormality is one of the
complications of chorionic villous sampling (CVS). The risk of
this complication depends upon the age of gestation; the risk
is greatest when the age of gestation is less than 9 weeks,
and is lowest when it is greater than 11 weeks.
Cordocentesis or Percutaneous Umbilical Blood Sampling
(PUBS) is used for rapid karyotype analysis, or when fetal
blood dyscrasias, such as fetal anemia and Rhesus
isoimmunization, are suspected. In the present case,
ultrasound has to be performed first to rule out the other
causes of MSAFP elevation beside genetic anomalies.
Serum estriol levels can be measured in pregnancy along with
MSAFP and beta-hCG (Triple test) as a screening tool for
chromosomal anomalies in the fetus.
Down syndrome: is the most common autosomal abnormality
among live births and the most common cause of mental
retardation in children. Most cases are caused by total trisomy
21, which typically arises from maternal meiotic
nondisjunction. Since advanced maternal age (defined as 35
years of age or older) is associated with an increased risk of
having offspring with Down syndrome, accurate prenatal
screening for the condition is in high demand. At this time,
the integrated test is considered the best overall screening
test for Down syndrome, with a detection rate of 85% and a
false positive rate of 1.2%. The integrated test is comprised
of ultrasound measurement of nuchal translucency thickness

at 10 weeks in combination with measurement of serum


markers from the first trimester (PAPP-A) and the second
trimester (alpha-fetoprotein, human chorionic gonadotropin,
unconjugated estriol, and dimeric inhibin-A); however, the
actual diagnosis of Down syndrome can only be made by
examining the fetal karyotype, which requires chorionic villus
sampling or, more commonly, amniocentesis.
BioPhysical Profile: BPP is a scoring system to evaluate baby's
well being. Its indicated when there is Decreased movement
or a non-reactive NST. It includes NST in addition to 4 things,
1-Fetal tone, 2-Movment, 3-Breathing(30/10min), 4-Amniotic
fluid inxed(5-20). Each has a score of 2, when present and 0
when absent. 8-10 is normal, and should be repeated once or
twice weekly, until term.In presence of OlygoHydramnions
(AFI<5) delivery is considerd. If score is 6 w/o OH,
Contraction stress test is ordered. If it gives non-reassuring
results then Deliver, if it gives suspicious results then repeat
the next day. If its 4 w/o OH and fetal lung are mature,
Delivery is considered. If lungs are not mature, steriods is
gven and BPP accessed w/i 24 hrs. If score is <4, deliver now.
Genetics: If the father is affected by an X-linked recessive
disease and the mother is not a carrier, none of their children
will be affected by the disease, although all their female
children will be silent carriers.
For autosomal-dominant diseases, the probability of the
disease for the child is close to 50% if one of the parents is
affected.
For autosomal recessive diseases, the probability of the
disease for the child is close to 25% if both parents are silent
carriers.
Fetal hydantoin syndrome presents with a small body size
with microcephaly, hypoplasia of the distal phalanx of the
fingers and toes, nail hypoplasia, low nasal bridge, hirsutism,
cleft palate and rib anomalies.
Fetal Alcohol Synd: Irritability, mild to moderate mental
retardation, hpoplastic maxilla, lng philtrum, thin upper lip
border and microcephaly, and epicanthal folds.

Pseudocyesis: Pseudocyesis is an imaginary pregnancy that


usually occurs in women having a strong desire to become
pregnant or in need for motherhood. It was suggested that
the depression caused by this need was behind the
occurrence of some hormonal changes similar to those of
pregnancy, and thus resulting in nearly all its symptoms and
signs. Patients may present with amenorrhea, enlargement of
the breasts and abdomen, morning sickness, weight gain,
changes of the uterus and cervix, sensation of fetal
movements, and even a positive urine pregnancy test.
Ultrasound, however, reveals a vacant uterus with normal
endometrial stripe and the pregnancy test is negative. The
woman may show an acute depression when no baby is born.
Abortions: Abortion is a spontaneous loss of pregnancy that
occurs before 20 weeks gestation. It is also defined as the
expulsion of a fetus, or an embryo, weighing 500g, or less.
Preterm birth is a spontaneous termination of pregnancy that
occurs between 20 and 37 weeks gestation.
In complete abortion, the whole conceptus passed through
the cervix. The cervix then closes, and pain and uterine
contractions subside. Ultrasonography shows an empty
uterus. Incomplete abortion implies the evacuation of some of
the fetal tissue while the rest is still retained in the uterine
cavity.
Threatened abortion is any hemorrhage occurring before the
20th week gestation, with a live fetus. The cervix is closed
and there is no passage of fetal tissue. Mild lower abdominal
pain may be noted. The fetal heart is active on Ultrasound.
25% of pregnancies have vaginal bleeding, to some extent, in
the first trimester. In half of these cases, the abortion will
actually occur.
Inevitable abortion manifests with vaginal bleeding, lower
abdominal cramps that may radiate to the back and
perineum, and a dilated cervix. Ultrasonography
demonstrates a ruptured or collapsed gestational sac with
absence of fetal cardiac motion. Incomplete abortion has the
same presentation except that there is an incomplete
evacuation of the conceptus. Ultrasonography reveals
endometrial debris.
Septic Abortion Is to be suspected in ALL abortions outside
hospital who come bcack with pain and vaginal discharge. Its
managed with cervical and blood sampling, IV antibiotics and

gentle suction cuertage.


Abruptio Placenta: Pt presents with vaginal bleeding,
ABDOMINAL PAIN, and uterine tenderness. The absence of
hemorrhage DOES NOT rule out this Dx.
DDX:Placenta Previa, absence of bleeding RULES OUT this dx.
Risk factors are:1-HT and preecclampsia, 2-Placental
abruption in previous pregnancy, 3-trauma, 4-short umbilical
cord, 6-COCAINE abuse.
AP is the mcc of DIC in pregnancy, which results from a
release of activated thromboplastin from the decidual
hematoma in to maternal circulation.
It can progress rapidly so careful monitoring is mandatory.
Once dx is made, large-bore IV , as well as Foley cathater is
inserted. Pts with AP in LABOR should be managed
aggressively to insure rapid vaginal delivery, since this will
remove the inciting cause of DIC and hemorrhage. Now if pt
is stable tocolysis with MgSO4 is considered, but remmeber
Ritordin is CI in pt with HT. Again, once we dx the next step is
Vaginal delivery with augmentation of labor if necessary. Now
if mother and baby are not stable or if there is CI, then
Emergency C-section is indicated. Now if there is Dystocia
( narrowing of the birth passage) then Forcepts can be used.
Placenta Previa: presents with painless vaginal bleeding in the
3rd trimester. Dx is with US, accuracy of 90%
transabdominally and 100% transvaginally. Pelvic exam is CI.
Risk factors are: multi parity, advanced age, multiple
gestation. If the bleeding continues, C-section has to be done
ASAP. Even if pregnancy is not term yet. In cases of extended
bleeding surgeon might discover A Placenta Acerta. Now if the
mother is stable and fetus is at term, scheduled C is choice.
However, until then pt has to be monitored closely. If both
mother and fetus are stable IM steriod are used to mature the
lungs. Forcepts are not used in complete PP.
In any case Vaginal delivery is not an option for any kind of
PP. If both mothe and fetus are OK and there is no more
bleeding and mother has access to close hospital then she can
be sent home and monitored at home until 36week, then
schedule c-section is done.
Pt with complete PP, after bleedng is controlled and both baby
and mohter are fine, Schedule Elective C-section.
Vasa Previa: Hemorrhage w fetal heart rate changes,
progresing from tachycardia to bradycardia to a sinusoidal

pattern. "Apt" test distinguishes maternal from fetal blood.


Mortality risk is 75%. Dx w transvaginal US in combination w
Doppler. VP necessitates emergency C section.
Preeclampisa/Eclampsia: A history of preeclampsia in the past
increases the risk of developing this complication during a
subsequent pregnancy. The risk is higher if the preeclampsia
presented earlier (age of delivery was less than 32-33
weeks), or if the patient has renal disease or chronic
hypertension.
Patients at term and with severe preeclampsia should be
promptly managed with hydralazine (or labetalol) to lower the
blood pressure, and MgSO4 to prevent progression to
eclampsia. Once initial stabilization has been obtained,
delivery should be carried out. For patients with mild
preeclampsia, vaginal delivery is the preferred mode of
delivery. For patients with severe preeclampsia, the preferred
mode of delivery has not been evaluated. Cesarean section
should be decided on an individual basis.
Patients with severe preeclampsia and who are remote from
term are best managed in a tertiary care center or by
referring them to obstetricians who specialize in the
management of high-risk pregnancies.
Patients with severe preeclampsia are at greater risk of
developing eclampsia. The first priority in patients with
eclampsia or postictal coma is respiratory and cardiovascular
resuscitation. Anticonvulsant medications can be administered
after placing two large-bore needles in the patient. The most
effective agent used in these cases is magnesium sulfate;
however, the most effective treatment to prevent further
complications is to accelerate delivery. Eclampsia can cause
several other complications besides seizures, such as
disseminated intravascular coagulopathy, acute renal failure,
hepatocellular injury, liver rupture, intracerebral hemorrhage,
etc. Magnesium sulfate prevents seizures, but it will not stop
the pathologic process.
So if Qs asks Which of the following is the most effective
strategy to decrease this patient"s risk for developing further
complications?
Ans: Speed vaginal delivery or termination of pregnancy
if the question asks about the next step in the management?
Magnesium sulfate, because the hemodynamic stability and

seizure control are important before attempting delivery.


So the bottom line is If the patient is in the third trimester,
especially in the last six weeks, termination of pregnancy is
advised in order to stop the pathologic process. There is no
pharmacologic therapy more effective than this intervention.
Anti-seizure prophylaxis in a patient with eclampsia has been
a topic of prolonged debate. Recently, some studies have
confirmed that magnesium sulfate is not only the best
anticonvulsant medication for patients with eclampsia, but it
is also the more effective agent to prevent further seizures.
Diazepam or phenytoin can be added to the therapy if
seizures persist, even though the use of diazepam is limited
because of depressant effects on the fetus.
In pregnant patients with a hypertensive crisis, either
hydralazine or labetalol are the antihypertensive drugs of
choice. Methyldopa is preferred for oral therapy in mild to
moderate hypertension in a pregnancy.
Retinal hemorrhage is considered to be an extremely ominous
sign of preeclampsia/eclampsia.
Eclampsia is diagnosed when unexplained convuslions occur
in the setting of preeclampsia.
HELLP syndrome is characterized by thrombocytopenia,
microangiopathic hemolytic anemia, and abnormal liver
function tests. Magnesium sulfate is the standard of care for
patients with severe preeclampsia and HELLP syndrome,
because it significantly reduces the risk of seizure and is
relatively safe. Although 50% of all cases of eclampsia occur
prior to term, the risk is significant within the first 24-48
hours after delivery. Magnesium sulfate infusion should cover
this period. Antihypertensive agents may be indicated to
control elevated blood pressure if it exceeds 160/105 mmHg.
Plasma exchange transfusion is useful in patients with
persistent HELLP syndrome.

#8

drtanvir

Dec 18, 2008 - 12:17 PM

Forum Senior

Preterm pregnancy: Respiratory distress syndrome (RDS) is a


common complication in preterm infants. It occurs when fetal
lung maturity has not been reached yet. Other complications
of preterm birth include intraventricular hemorrhage, sepsis,

Topics: 23
Posts: 118

necrotizing enterocolitis and kernicterus. The mortality rate in


preterm infants is greatly influenced by the gestational age.
For instance, it is similar to that of full-term infants when
birth occurs at 36 weeks and attaining 50% when it occurs at
24 weeks.
The patient in advanced stage of labor should be managed
more aggressively and tocolysis has to be instituted at once.
Magnesium sulfate is now the drug of choice for tocolytic
therapy because of its fewer side effects such as feeling of
warmth and flushing. However, it is not FDA approved for this
use. It is particularly useful in the presence of
contraindications to beta 2 agonists, such as diabetes and
heart disease. It acts by competing with calcium ions for
entry into the cell.
Among tocolytic agents, Ritodrine (beta 2 agonist) is the only
agent approved by FDA for management of preterm labor.
Terbutaline is also widely used in the US and exists in the
oral, subcutaneous and intramuscular forms. Most side effects
of beta-agonist result from their concurrent b1 activity, and
include increase in heart rate, rise in systolic pressure,
decrease in diastolic pressure, chest pain secondary to MI,
and arrhythmia. Beta agonists may also cause fluid retention
secondary to decrease in water clearance, which when added
to the tachycardia and increased myocardial work, may result
in cardiac failure. In addition, they increase gluconeogenesis
in the liver and muscle, resulting in hyperglycemia, which
increases insulin requirements in diabetic patients. The
passage of beta-agonists through the placenta does occur and
may be responsible for fetal tachycardia, as well as
hypoglycemia or hyperglycemia at birth.
Other tocolytic gents include prostaglandin synthetase
inhibitors, such as indomethacin, and calcium channel
blockers. Indomethacin is quite effective but it can result in
oligohydramnios and premature closure of ductus arteriosus.
It also decreases fetal renal function and increases the risk of
necrotizing enterocolitis and intracranial hemorrhage. Calcium
channel blockers are promising agents and nifedipine may
gradually replace I.V. b-agonists. Its only side effects are
headache, hypotension, tachycardia and cutaneous flushing.
Tocolytic therapy does not decrease the incidence of preterm
births but prolong gestation, increasing thus, birth weight and
decreasing complications related to prematurity.

Contraindications include chorioamnionitis, pre-eclampsia,


placenta abruptio, severe placenta previa bleeding, fetal
demise and fetal anomalies incompatible with life.
Cervical cultures should also be performed to detect urinary
or genital infections, as well as the presence of group B
streptococcus. It is reasonable to administer prophylactic
antibiotic therapy since it has been proven to prolong
pregnancy even in the absence of ruptured membranes.
Steroids are also necessary at this stage of pregnancy. When
administered between 24 and 34 weeks gestation, they
accelerate fetal lung maturity and thus decrease the incidence
of RDS. They also reduce mortality and the incidence of
intraventricular hemorrhage.
Post-term pregnancy: Post-term pregnancy is defined as a
pregnancy age more than 42-weeks gestation. Perinatal
mortality is 2-3 times higher in post-term pregnancies and it
is related most commonly to post-maturity syndrome, which
occurs consequently to aging and infarction of the placenta.
Post-mature infants typically have a loss of subcutaneous fat,
long fingernails, dry and peeling skin, and abundant hair. In
70-80% of cases, fetuses are not affected by placental
insufficiency and continue to grow past 42-weeks gestation,
resulting in macrosomia. The cause of post-term pregnancy is
unknown; however, some associated syndromes have been
noted such as anencephaly and trisomy 18.
In terms of diagnosis of post-term pregnancy, the importance
of accurate dating cannot be stressed enough. In fact 20-30%
of pregnancies have uncertain dates, which may mislead to
another diagnosis or cause it to be overlooked.
The management of post term pregnancy is principally based
on the well being of the fetus. The non-stress test and
biophysical profile should be performed twice weekly and if
there is oligohydramnios or if spontaneous decelerations are
noted, delivery has to be accomplished. If on the contrary,
those parameters are reassuring, as in this case, labor should
not be induced unless the cervix is favorable, the infant is
macrosomic or in the presence of obstetrical indications for
delivery. If the pregnancy is more than 43 weeks, delivery is
mandated. If the pregnancy is more than 42 weeks, the
cervix is favorable and fetal head is into the pelvis labor
should be induced. Patients with uncertain dates should be

managed expectantly as long as fetal assessment is


reassuring, and the possibility of preterm pregnancy should
be considered as much as that of post-term pregnancy.
Expectant management is more appropriate since fetal heart
tracing is reassuring and there is no oligohydramnios.
Moreover, the cervix is not favorable for induction of labor.
The patient does not have oligohydramnios (AFI of 5 or
more), so amnioinfusion is not necessary.
C. Section is indicated in the presence of signs of fetal
distress. The fetal heart activity of this fetus is normal.
Fetal lung maturity is not a concern here since it occurs in
preterm not post-term infants. The respiratory condition most
frequent in post term fetuses is meconium aspiration.
Premature rupture of membranes (PROM): Premature rupture
of membranes (PROM) is a rupture that occurs before labor at
any time of gestation. Preterm PROM (PPROM) is the rupture
of membranes that occurs before term, whether there are
uterine contractions or not. The origin of PROM or PPROM is
not clear yet. Several etiologic factors, however, have been
supposed to contribute to its occurrence. Such factors include
infection, abnormal membrane physiology, incompetent cervix
and nutritional deficiencies of copper and ascorbic acid. The
diagnosis of rupture of membranes is mainly clinical. The
patient usually complains of either a gush or continual
leakage of clear fluid from the vagina. Differential diagnoses
include urinary incontinence, vesicovaginal fistula and
leukorrhea. On examination, amniotic fluid may be noted in
the vagina or leaking through the cervix when Valsalva
maneuver or slight fundal pressure is applied. When the
patient is not in labor, and whether the pregnancy is at term
or not, the hand of the examiner should not be inserted into
the vagina because of the increased risk of infection, and the
examination should be performed with a sterile speculum. The
amniotic origin of the fluid is confirmed by Nitrazine paper,
which turns blue when in contact with amniotic fluid. It can be
confirmed by demonstration of arborization, or ferning
when the fluid is air-dried on a slide and examined at low
magnification. In the case of PPROM, amniotic fluid sampling
to measure fetal lung indices is mandatory. Also, ultrasound
examination should be performed to detect fetal anomalies,
determine gestational age and measure amniotic volume,

which will all influence the management decision. In the case


fetus with a severe congenital anomaly incompatible with life
labor should be allowed to proceed.
Intra Uterine Fetal Demise (IUFD): Intra Uterine Fetal Demise
(IUFD) is the death of a fetus in utero that occurs after 20
weeks gestation and before the onset of labor. It can be
caused by a multitude of conditions such as hypertensive
disorders, diabetes mellitus, placental and cord complications,
antiphospholipid syndrome, congenital anomalies, and fetal
infections (TORCH, Listeria). The cause remains, however,
unknown in 50% of cases. A search should be undertaken to
determine the cause after the first episode of IUFD. The work
up should include: serology testing and cultures of TORCH,
Listeria and Parvovirus infections, an autopsy, complete body
radiographs and chromosomal studies of the fetus, testing for
anticardiolipin antibodies and Kleihauer-Betke test to detect a
fetomaternal hemorrhage. It is very important to try to
diagnose the cause of the demise after the first episode and
to correct it, in order to prevent recurrence in the following
pregnancy. IUFD is suspected when the patient reports the
disappearance of fetal movements, a decrease or stagnation
in uterine size, and/or when fetal heart sounds are no longer
heard. The most appropriate test to confirm this diagnosis is
Real Time Ultrasonography, which will demonstrate a lack of
fetal movement and absence of fetal heart activity. Beta-hCG
serum levels are not useful because they remain usually
elevated because of an ongoing placental production. After
the diagnosis is confirmed, the coagulation profile has to be
determined to detect an eventual disseminated intravascular
coagulation, which is a serious complication of IUFD, early in
its course.
IUGR: IUGR is defined as birth weight below the 10th
percentile for a given gestational age, and refers to fetuses
and neonates whose growth potential has been restricted by
pathologic processes in utero. These fetuses are particularly
prone to problems such as meconium aspiration, asphyxia,
polycythemia, hypoglycemia, and mental retardation.
Once IUGR diagnosed, fetal well-being has to be closely
monitored with NST and BPP twice weekly. The mother can
also contribute to this monitoring by assessment of a kick
count.

Decision to deliver depends on fetal well-being and lung


maturity. Delivery is usually indicated at 34-weeks or later, or
at any gestational age when fetal lung maturity is reached. If
there is an associated oligohydramnios, delivery should be
strongly considered. In addition, IUGR fetuses are more
susceptible to asphyxia, and the threshold to perform a C.
section should be lower than that of normal fetuses. In fact,
some signs on fetal heart tracing that may be considered
reassuring in a normal fetus, are ominous in a fetus with
IUGR and may indicate distress.
At birth, neonates with IUGR are very prone to hypothermia,
and hypoglycemia. Respiratory distress syndrome also
frequently occurs because of reduced surfactant synthesis and
release caused by asphyxia and resultant acidosis.

#9

drtanvir

Dec 18, 2008 - 12:18 PM

Forum Senior

Labor: Labor is defined as the progressive cervical


effacement, dilatation, or both resulting from uterine
contractions, which occur at least every 5 minutes and last 30
to 60 seconds.
Labor progresses through four stages:
The first stage extends from the onset of labor until full
dilation of the cervix, and includes two phases: a latent
phase, during which dilation progresses at a slow rate until
reaching 2-3cm, followed by an active phase, during which
the dilation is more rapid. The length of the latent phase is
highly variable but is considered prolonged when it exceeds
20 hours in the primiparous and 14 hours in the multiparous.
The progression of the active phase is evaluated by the rate
of cervical dilation. At this phase, the cervix normally dilates
at a rate of at least 1cm/hr in the primiparous and 1.2cm/hr
in the multiparous.
The second stage of labor extends from complete dilation of
the cervix to delivery of the baby. It usually lasts 30 min to 3
hours in the primiparous, and 5 to 30 minutes in the
multiparous. The third stage of labor starts with the delivery
of the baby, and ends with the delivery of the placenta. The
fourth stage extends from delivery of the placenta until 6
hours postpartum. The mother should be closely observed

Topics: 23
Posts: 118

during this stage because of the risk of postpartum


hemorrhage.
Prolonged latent phase can be caused by hypertonic uterine
contractions, hypotonic contractions, or premature or
excessive use of anesthesia or sedation. Hypertonic
contractions, although intense, are ineffective. They are more
painful and are associated with increased uterine tone.
Hypertonic activity of the uterus usually responds to
therapeutic rest with morphine sulphate or an equivalent
drug. Hypotonic contractions are less painful and are
characterized by an easily indentable uterus during the
contraction. Sometimes, patients diagnosed with prolonged
latent phase may actually be still in false labor. Contractions
of false labor are usually painless and sporadic, but can be
rhythmic, occurring every 10 to 20 minutes. Their main
characteristic, however, is that they are not accompanied with
cervical changes. Patients with hypocontractile dysfunction
are best treated with a diluted infusion of oxytocin.
Anesthesia may reduce uterine activity if administered in the
latent phase (such as in this case). In the active phase, it has
either the opposite effect, or no significant effect. The only
treatment when it is the cause of a prolonged latent phase is
to allow the responsible drug to be evacuated; the uterus
usually resumes its normal activity afterwards.
Twin pregnancy: Recent studies have indicated that if the fetal
heart rate is reassuring, the second twin does not have to be
delivered within a fixed time frame after the first twin.
Expectant management suffices for the spontaneous
deliveries of most twins, but electronic fetal monitoring and
ultrasound are of significant help in promptly recognizing fetal
distress when it does occur.
Once the first twin is delivered, the positioning and heart rate
of the second twin must be assessed with ultrasound. If labor
has halted, oxytocin should be administered.
Arrest disorders: Hypotonic uterine contractions is the most
common cause of arrest disorders in the active phase of labor.
Patients with such conditions should undergo an
augmentation of labor using amniotomy and/or oxytocin
infusion.
False Labor: Characterized by painless and irregular

contractions for 5hrs or more. In the last month these


contractino may become rhythmic occuring every 10-20
minutes mimicking contraction of real labor. The main
characteristic is however they are not accompanied with
progressive cervical changes, so cervix is closed shut. All the
pt needs is reassurance.
Deccelereaions: Early decelerations defined as decrease in
fetal heart rate by 15 beats/sec from baseline for at least 15
seconds, occuring at the same time as the uterine
contraction.
Fetal sleep presents with decreased long-term variability.
Fetal cord compression presents with variable decelerations.
Such decelerations are not related to uterine contractions, are
variable in form and may be non-repetitive. The first step in
the presence of variable deceleration is oxygen administration
and change in maternal position to the left or right side.
Placing the patient in the Trendelenburg position and
elevating the presenting part may be tried thereafter, if the
anomaly is persistent. Amnioinfusion can be used when
variable decelerations occur after the membranes have
ruptured, in order to relieve cord compression. However,
regardless of membranes status, it is not a first-line
measure and the change in position plus mask oxygen is
more appropriate.
Uteroplacental insufficiency presents with late decelerations.
Intrauterine infection presents with tachycardia, which may
be associated with other signs of fetal distress.
Scalp pH is performed if the abnormal fetal heart pattern
persists after all initial measures (i.e.; position change, mask
oxygen and discontinuation of oxytocin) have been tried.
Cesarean section is indicated when fetal distress is confirmed.
Preterm labor: Antenatal corticosteroid therapy has been
proved to be effective in reducing perinatal morbidity and
mortality associated with preterm labor. It reduces the risk of
infant respiratory distress syndrome by stimulating
phospholipids synthesis (surfactant!) and accelerating
morphologic development of the lungs. Besides that,
antenatal corticosteroid therapy appears to reduce the risk of
intraventricular hemorrhage in infants. It should be given to
any pregnant woman from 24 to 34 weeks of gestation with

intact membranes at high risk for preterm delivery. Two


regimens of therapy are available: betamethasone and
dexamethasone (some authors believe that betamethasone is
preferred over dexamethasone). Intramuscular administration
of steroids provides stable and predictable concentration of
the drug in the blood that is required to achieve desired fetal
effects. Intravenous administration results in peaks and
troughs in the blood concentration of the drug and, therefore,
is not recommended. One study that compared an oral
regimen to intramuscular administration of dexamethasone
found increased risk of intraventricular hemorrhage and
sepsis in the oral dexamethasone group.
Shoulder dystocia: Shoulder dystocia is commonly defined as
a failure of the fetal shoulders to pass through the maternal
pelvis once the fetal head has been delivered. It is diagnosed
when the anterior shoulder cannot be delivered with mild,
downward pressure. Risk factors for the development of
shoulder dystocia include macrosomia, maternal diabetes
mellitus, operative vaginal delivery, shoulder dystocias in
previous deliveries, postdate pregnancies, male fetal gender,
advanced maternal age, obesity and excessive weight gain,
and disproportion between the fetal shoulders and maternal
pelvis. However, more than 50% of cases of shoulder dystocia
are not associated with any known risk factors. When
shoulder dystocia occurs, appropriate support staff (eg,
nursing, anesthesia, obstetrics, pediatrics) should be
summoned. The mother should be told not to push while
attempts are made to reposition the fetus. Suprapubic
pressure directed downward and laterally should then be
applied by an assistant. If that fails to deliver the anterior
shoulder, then typically the McRoberts maneuver is
attempted, though many other techniques work as well (eg,
Rubin maneuver, Woods screw maneuver, delivery of posterior
arm).
The McRoberts maneuver requires that two assistants grasp
both of the mothers legs and flex the thighs back against
her abdomen. This maneuver has been shown to relieve the
shoulder dystocia in 42% of patients. Before the maneuver is
implemented, however, the mother should be told to stop
pushing until everything is in place.
The Zavanelli maneuver replaces the fetal head in the pelvis
before performing a cesarean section. It is generally accepted

that the physician has up to seven minutes to deliver a


previously well-oxygenated infant before there is an increase
in the risk of damage due to asphyxia. Therefore, the
Zavanelli maneuver is normally employed when other
methods have failed and the "safe" period of seven minutes is
dwindling.
Breech presentation: If prior to 37 must be left alone. After
that External cephalic version may be attempted PRIOR TO
onset of labor, given no CI (Hypertension).
Post partum hemorrhage (PPH): The most common cause of
post partum hemorrhage (PPH) is uterine atony, and the first
step in the management of all patients with PPH is to do a
pelvic examination to identify retained placental fragments. If
the retained placental products are not identified, manual
uterine massage should be started. Uterine massage
stimulates the uterus, uterine contractions start, and bleeding
stops.
If bimanual uterine massage fails to control bleeding,
uterotonic drugs (like IV oxytocin) are used to control
bleeding.
If bleeding does not stop with the medical measures, surgical
measures are taken. Uterine artery ligation is one of the
surgical measures for the treatment of PPH.
Uterine Rupture: Presents with intense abdominal pain asso w
vaginal bleeding, ranging from spotting to massive
hemorrhage.
Risk in transverse line is 0.5% and in Vertical its 5.0%. If pt
does not want any more children, total hysterectomy is the Tx
of choice. If she wants more kids then Debridment and
closure is indicated. Difficult to ddx from abrupta placenta. UR
is preceeded by agitation, rapid breathing, tachycardia.
Postpartum endometritis: It is a polymicrobial infection of the
decidua (the pregnancy endometrium) characterized by fever,
uterine tenderness, foul-smelling vaginal discharge, and
leukocytosis. As the infection is often produced by both
aerobes and anaerobes from the genital tract, any treatment
regimen must include broad-spectrum antibiotics that also
cover beta-lactamase producing anaerobes. At this time, the

gold standard of treatment for endometritis is clindamycin


and gentamicin.
Metronidazole is contraindicated in breastfeeding mothers.
Endometritis occurs after 3% of vaginal births, but after 1530% of cesarean deliveries (especially those performed after
labor commences or after the rupture of membranes). Studies
show that the most important risk factor in the development
of endometritis is the route of delivery. Several causes
contribute to this finding, including contamination of the
uterine cavity, prolonged rupture of membranes, and
presence of sutures or other foreign objects.
Chorioamnionitis: Chorioamnionitis is the infection of the
amniotic fluid surrounding the fetus. It is frequently
associated with preterm or prolonged rupture of membranes
but can also occur with intact membranes. Patients with
chorioamnionitis exhibit fever greater than 38C(98.7F),
uterine tenderness and irritability, elevated WBC count and
fetal tachycardia. Maternal infection of any origin can cause
fever, elevated WBC count as well as fetal tachycardia. So, a
careful examination to detect any other foci of infection is
necessary. Fetal tachycardia may also be congenital or
consequent to beta 2-agonist administration to the mother,
for example for tocolysis. Elevation of maternal WBC also
occurs at the onset of labor or can be caused by corticosteroid
administration. Amniotic fluid cultures are the gold standard
for diagnosis if any doubt remains.
Once chorioamnionitis is diagnosed, broad-spectrum antibiotic
therapy should be instituted, but not before samples for
culture are taken. Ampicillin and gentamicin are the drugs of
choice. Labor should thereafter be induced or augmented if
contractions have already begun. If the cervix is unfavorable
and fetal heart tracing is alarming, a C- section may be
necessary.

#10

drtanvir

Dec 18, 2008 - 12:19 PM

Forum Senior

Diabetes: All pregnant patients are generally screened for


gestational diabetes between the 24th and 28th weeks of
pregnancy using the 50 gm glucose tolerance test. This test

Topics: 23
Posts: 118

does not require fasting. Blood glucose levels are checked one
hour after the ingestion of 50 gm of glucose. Patients with
blood glucose values of 140 mg/dL or higher should be
subjected to a 3-hour glucose tolerance test after the
ingestion of 100 gm of glucose on a fasting state. Two or
more blood glucose values greater than 105, 190, 165 and
145 mg/dL at 0, 1, 2 and 3 hours, respectively, are diagnostic
of gestational diabetes. Some workers have proposed lower
cutoff values for the diagnosis of gestational diabetes. The
American Diabetes Association is recommending the use of a
75-gm glucose tolerance test, with different cut-off values, for
use in non-pregnant women and for diagnosing gestational
diabetes.
The recommended fasting blood glucose values in pregnant
diabetic patients should range between 60-90 mg/dL, and
postprandial blood glucose values should be less than 120
mg/dL. NPH in combination with regular or lispro insulin is
generally recommended if diet and exercise are not able to
adequately control the blood sugar. Oral hypoglycemics are
not indicated in pregnant patients. The use of glargine insulin
is not considered safe during pregnancy.
UTI: In pregnant patients with signs and symptoms
suggestive of urinary tract infection, begin empiric antibiotic
therapy immediately with cephalexin, amoxicillin, or
nitrofurantoin for a period of 3-7 days. Some experts also
advocate obtaining a urine culture to allow for later
modification of the treatment regimen based upon pathogen
sensitivity and patient response to pharmacotherapy.
Trimethoprim-sulfamethoxazole is not advised in the first or
third trimester of pregnancy.
Acute pyelonephritis is usually characterized by costovertebral
angle tenderness, fever, chills, dysuria, nausea, vomiting, and
respiratory discomfort. Pregnant women are particularly
susceptible to experiencing adverse outcomes secondary to
pyelonephritis (eg, septic shock syndrome, preterm birth, low
birth weight). Traditionally, pyelonephritis in the pregnant
woman is treated with hospitalization and intravenous
antibiotics such as ceftriaxone or ampicillin and gentamicin
until she is afebrile for 24-28 hours and experiencing an
improvement in symptoms. Some providers will consider
outpatient treatment if the pyelonephritis is otherwise

uncomplicated. Whether the treatment is provided on an


inpatient or outpatient basis, the treatment course should last
for 10-14 days. For the remainder of the pregnancy, low-dose
antibiotic prophylaxis with nitrofurantoin or cephalexin should
be accompanied by occasional urinary evaluation for infection.
If the symptoms of pyelonephritis do not respond to antibiotic
therapy within 48 hours, then a renal ultrasound to assess for
perinephric abscess or renal calculi should be performed.
Repeat: Traditionally, pyelonephritis in the pregnant woman is
treated with hospitalization and intravenous antibiotics such
as ceftriaxone or ampicillin and gentamicin until she is afebrile
for 24-28 hours and experiencing an improvement in
symptoms. She should then be discharged on oral antibiotics,
with the entire treatment course lasting for 10-14 days.
Ultrasound is the test of choice for suspected nephrolithiasis
in a pregnant patient.
Helical CT scan is the test of choice for suspected
nephrolithiasis; however, a CT scan is associated with
radiation exposure, and is not the best test in pregnant
patients.
Asymptomatic bacteriuria must be treated promptly in the
pregnant female as it can quickly progress to cystitis and
pyelonephritis. Although Streptococcus agalactiae (Group B
Streptococcus) is a less common cause of cystitis and
pyelonephritis in pregnancy than Escherichia coli, its presence
is indicative of significant genital colonization with Group B
Streptococcus, which is associated with an increased risk of
preterm labor and premature birth.
positive urine culture for Group B Streptococcus in a pregnant
female should be treated immediately with a 10-day course of
antibiotics. The antibiotic may either be penicillin G or
cephalexin; both antibiotics are considered safe for the fetus.
After the treatment course is completed, it is important to
document that the patients urine is sterile.
Thyroid disease: As soon as pregnancy is confirmed in
hypothyroid patients who are receiving hormone replacement
therapy, the dose of L-thyroxine should be increased. During
pregnancy, the requirement in levothyroxine increases by 25
to 50%. The causes of the increased requirement for Lthyroxine during pregnancy include increases in thyroxinebinding globulin (TBG) and the volume of distribution of T4,

and an increase in body mass. The serum TSH level is a


sensitive marker of the hypothyroid state, and should be
checked each trimester. The dose of L-thyroxine should be
adjusted accordingly.
Due to an increase in serum concentrations of TBG, pregnant
women have high levels of T3 and T4. In order to determine if
a patient has true hyperthyroidism, free T4, total T4 and TSH
levels need to be determined.
In order to diagnose a pregnant woman with hyperthyroidism,
she must meet the following criteria:
1. high serum free T4
2. serum TSH value < 0.01 mU/L
On the other hand, gestational transient thyrotoxicosis (GTT)
is diagnosed if the woman meets the following criteria:
1. mildly increased free T4
2. slightly decreased TSH levels at the end of the first
trimester
GTT is a medical condition that occurs approximately in 10%
of women. It usually presents between 8 to 11 weeks of
gestation, and does not occur after 14 weeks. It is due to the
elevated levels of human chorionic gonadotropin (hCG), which
has thyroid-stimulating properties.
Sickle-cell disease: Patients with sickle-cell disease are at a
very high risk for complications during pregnancy. The
possible complications include acute crisis, endometrial
infection, pyelonephritis, and thromboembolic events. Up to
46% of sickle-cell patients have complications during
pregnancy. Such patients therefore need a safe and very
effective method to prevent undesirable pregnancies.
Therefore, patients with sickle-cell disease need a safe and
reliable method for contraception to prevent undesired
pregnancies. The use of oral contraceptive pills in patients
with sickle-cell disease is controversial despite the fact that
no studies show any associated increased side effects with
OCP use. Depot medroxyprogesterone appears to be safe and
effective in preventing pregnancies in patients with sickle-cell
disease.
Syphilis: Any pregnant woman that has positive treponemal
tests should be considered infected until proven otherwise.
Untreated syphilis is associated with a very high prevalence of
adverse fetal outcomes (up to 80%), including stillbirth,

neonatal death, and mental retardation. Appropriate therapy


should be promptly instituted. Penicillin remains the drug of
choice (gold standard) for the treatment of syphilis. In
patients with a penicillin allergy, penicillin desensitization is
recommended. It is typically accomplished using incremental
doses of oral penicillin V.
Cardiology in pregnancy: Women who have congenital heart
disease and become pregnant are at risk for significant
maternal and fetal deterioration. Pregnancy is associated with
a number of physiologic changes, including expansion of
intravascular volume, changes in the cardiac output, and
systemic vascular resistance. The risk of complications in such
patients is even higher if the patient has Eisenmenger
syndrome (i.e., severe pulmonary hypertension and shunt
reversal). Pregnancy in these patients is associated with a 3050% risk of mortality. Majority of the maternal deaths occur
during the first week after delivery; however, death can also
occur during gestation, labor, or delivery since the sudden
drop in systemic vascular resistance after delivery increases
the right-to-left shunt, which may precipitate maternal
cyanosis. Unfortunately, a sudden increase in systemic
vascular resistance can also lead to adverse effects due to the
decreased cardiac output, which can cause syncope.
Furthermore, there is an associated higher incidence of
spontaneous abortions, pre-term delivery, and fetal mortality
in these patients. Due to these reasons, all pregnant patients
with Eisenmenger syndrome should be advised to have an
elective termination of pregnancy.
Anticoagulation: The goal is to prevent thrombotic or embolic
complications in the pregnant patient, while avoiding fetal or
maternal harm due to anti-thrombotic agents. The most
common indications are the presence of mechanical prosthetic
heart valves or the history of venous thromboembolism.
Because warfarin is teratogenic and can freely cross the
placental barrier, a patient on anticoagulation therapy who
plans to get pregnant should replace warfarin with
subcutaneous unfractionated heparin or low-molecular-weight
heparin in the first trimester.
Pruritus: Pruritus is a very common complaint during

pregnancy. It is reported to affect up to 20% of pregnant


women. The symptom may be a manifestation of pregnancyassociated dermatosis, although there is usually no pathologic
process present in most cases. Common pruritic locations are
the scalp, anus, vulva, and abdomen (during the third
trimester). Pregnancy-induced pruritus may be related to
dermographism or urticaria, which are common in the last
half of pregnancy. Topical steroids, antihistamines, oatmeal
baths, emollients, and UVB are used to treat the condition.
Herpes gestationis (also called pemphigoid gestationis) is an
uncommon blistering dermatosis that is associated with
pregnancy. It typically starts during the second or third
trimester, or postpartum period. It may first manifest as
abdominal pruritis, a relatively common benign condition
during pregnancy, with later development of a rash. The rash
is localized around the umbilicus, and is characterized by
papules, urticarial plaques, and vesicles, although bullae may
also form. Contrary to its name, herpes gestationalis is not
caused by a viral infection, and is believed to be an
autoimmune disorder.
Herpes gestationis is an autoimmune disease of pregnancy.
Autoantibodies are detected in the skin as well as in the
circulation. Corticosteroids are the mainstay of therapy. In
early and mild cases, topical mid-potency steroids (e.g.,
triamcinolone) are used. In more advanced cases or if topical
steroids are not effective, systemic steroids are employed.
Note: Topical steroids and antihistamines are widely used to
treat many pregnancy-associated dermatoses, including
papular urticarial papules and plaques of pregnancy (PUPPP);
therefore, these are typically the correct choices for questions
asking about the treatment of pregnancy-associated
dermatoses!

#11

drtanvir

Dec 18, 2008 - 12:20 PM

Forum Senior

Rh incompatibility: Hemolytic disease of the newborn due


to Rh-incompatibility is possible only when the mother is Rhnegative and the father is Rh-positive.
Hemolytic disease is very unlikely in the first pregnancy, as
the mother is not sensitized. The mother becomes sensitized

Topics: 23
Posts: 118

as a result of fetomaternal hemorrhage at or near the end of


pregnancy. This risk of sensitization can be reduced by a
RhoGAM injection within 72 hours of the delivery.
Anti-D immune globulin should be administered
intramuscularly at 28 weeks gestation to all Rh (D)
negative women if no anti-D isoimmunization is detected by
antibody screening. Antenatal prophylaxis is not necessary
only if the father of the child is known with certainty to be Rh
(D) negative . Antenatal prophylaxis helps to reduce
significantly the risk of Rh (D) isoimmunization during
pregnancy. Peripartum anti-D immune globulin is also
indicated to reduce the risk of isoimmunization due to
fetomaternal transfusion during the delivery. Early antenatal
anti-D immune globulin prophylaxis is indicated in case of the
events and procedures that can increase the risk of
isoimmunization (e.g., spontaneous abortion, ectopic
pregnancy, and amniocentesis). Otherwise, the risk of
isoimmunization before the 28th week of pregnancy is very
low.
Failure to adjust the dose of anti-D immune globulin after
events that are associated with excessive feto-maternal
hemorrhage (e.g., placental abruption) may result in maternal
alloimmunization.
Events that are associated with feto-maternal hemorrhage
(such as placental abruption) may require adjustments in the
dosage of anti-D immune globulin; therefore, the presence
and the amount of feto-maternal transfusion should have
been determined in this patient during her first pregnancy.
The rosette test is a qualitative test that helps determine the
presence of feto-maternal hemorrhage. If negative, the
standard dose of anti-D immune globulin should be
administered. If positive, the amount of hemorrhage can be
evaluated using Kleihauer-Betke stain or fetal red cell stain
using flow cytometry, and the dose of anti-D immune globulin
should be corrected accordingly.
Pregnancy Facts:
Screening for gourpB strep should be done 36-37 week
gestation and positive cases should be tx with Penicilline G
during labor, even in the absence of risks.
Low back pain is very common in thrid trimester. Its caused
by lumbar lordosis and relaxation of ligament to the joints.

Both Graves dis and Migrain will improve in Pregnancy.


Excessive use of oxytocin may cause water retention (acts
like ADH), hyponatremia and seizures (water intoxication).
ACE inhibitors and Oral hypoglycemic agents (Glibenclamide)
are CI in pregnancy. Stop them and give Insulin for DM, and
Methyldopa(most comonly used), Hydralazine and Labetalol
for HT control in pregnancy.
Pregnancy hasa protective efect on both MS and PUD.
In pregnancy both BUN and Creatinin are decreased to half of
prepregnancy levels. Amoxicilin has no effect on them.
Asymptomatic Bacteriuria of Pregnancy, increases risk of
developing cystitis and pyelonephritis. E coli is the cause 70%
of the times. tx is 7-10 days of Nitrofurantoin, Ampicillin or
first gen Cephalos.
Neonates of pts with graves dis treated with with surgery are
at risk for Thyrotoxoicosis.because of the passage of throid
stimulating immunoglobin across the placenta.
Hypotension is a comon se of epidural anesthesia. The cause
of hypotension is blood redistribution to the lower extremities
and venous pooling.
In pregnancy its recommended to CONTINUE excercise as you
were doing before, like an aerobic instructor.
Edema of lower extremities (Bilateral) in pregnancy is most
commonly a benign problem. Pre-eclampsia should be
suspected if the edema is associated with hypertension or
proteinuria, no need to do ECG or DVT (presents unilateral
and fever).
Oxytocin is like ADH so it causes water retention and Water
toxicity due to decrease in Na concentration ( 123 ). So it
could cause seizure post partum.
Screening cultures should be performed at week 36-37 and
positive cases should be treared with IV penicillin therapy
during labor to prevent the new born from getting infected.
If hypertension sets in before 20 weeks, its either Mole or
CHRONIC hypertension. If it sets in after 20 weeks, its either
Preeclampsia (Proteinuria, >300mg) or TRANSIENT
hypertension (not accompanied by proteinuria, <300mg).
Antibodies to ABO antigens belong to IgM antibody class so
they dont cross placenta and hence mother and baby can
have different blood type. But Anti-D antibodies that are
responsible for Rh alloimmunization belongto IgG class and do
cross placenta.

All pregnant drivers should wear seatbelts while driving. Seat


belts have been shown to improve both fetal and maternal
outcomes in automobile accidents. Pregnant drivers who do
not wear seat belts while driving are more likely to have
excessive internal bleeding and fetal death if they are
involved in a car accident. The current recommendation is
that both lap and shoulder straps should be used. The lap
strap should be placed under the abdomen, and the shoulder
strap should be placed diagonally across the chest and
abdomen.
Adolescents pregnancy: Adolescents are at increased risk of
adverse pregnancy outcomes. These adverse outcomes
include increased perinatal mortality, preterm delivery and
premature and low birth weight infants. It is unclear whether
these outcomes are due to socioeconomic factors (e.g., low
education and low income) or biologic immaturity. A recent
study demonstrated unfavorable prognosis in the adolescent
pregnant women compared to the pregnant women aged 2024, even after adjusting for socioeconomic risk-factors;
therefore, biologic immaturity may be an important
contributor to the increased risk of teenage pregnancies.
Moreover, the children of adolescent mothers may be at
increased risk of severe socioeconomic outcomes, including
future cognitive disorders. Unlike late pregnancies, adolescent
pregnancy does not increase the risk of congenital
malformations.
Exposure to the low-energy electromagnetic fields that are
generated by video displays, power lines, and electronic
devices has not been demonstrated to be dangerous to a
fetus.
Ethics: Expectant teenage mothers are legally entitled to give
consent for procedures related to the management of their
pregnancies, including the performing of cesarean sections.
Adolescents do not need consent from their parents for
contraceptive services. Barrier methods should be used by all
sexually active adolescents even if an additional method of
contraception is being used.
Most states have implemented laws that allow for a physician
to provide certain types of medical care to adolescents
without parental consent. Elective abortions are typically

protected under these laws. Therefore, a teenaged girl would


have the authority to provide legal consent for an abortion.
When deciding ethical issues such as this, it is important to
focus on the principles of patient autonomy and beneficence.
Patients have the right to seek, accept, or refuse care. With
the mentally disabled, it may be impossible to obtain
informed consent, but at the least simple assent should be
strenuously pursued. If sterilization is not wanted, it should
not be performed against the patients will. Involuntary
sterilization violates a womans right to privacy, her
reproductive rights, and her bodily integrity.
Lesbian women often conceal their sexual orientation from
their primary care physicians. Recent developments and
legislations regarding same sex marriages have encouraged
more homosexual couples to "come forward" or reveal their
sexual orientation. It is important for primary care physicians
to be sensitive and knowledgeable to their special needs and
concerns.

dr_puma

Dec 18, 2008 - 9:23 PM

Forum Junior

nice,thanks

#12

Topics: 141
Posts: 1549

drtanvir

Dec 20, 2008 - 6:51 AM

Forum Senior

Puma
U r always supportive.
Thanx

Topics: 23
Posts: 118

#13

sukhs

Dec 20, 2008 - 10:57 AM

Forum Senior

Hey Tanvir..Are these from the USMLEWORLD??

#14

Thanks a lot

Topics: 18
Posts: 184

drtanvir

Dec 21, 2008 - 7:07 AM

Forum Senior

These are from different MCQs sources including UW.

#15

Topics: 23
Posts: 118

Karime

Dec 21, 2008 - 7:22 AM

Sun and Shield

I think they are good for Step 1 as well, right?

#16

Thanks!

Topics: 27
Posts: 2493

drtanvir

Dec 22, 2008 - 7:44 AM

Forum Senior

Epilespsy: Although anti-epileptic drug use during pregnancy


is associated with an increased risk of congenital
abnormalities, over 90% of women with epilepsy have normal
pregnancy. There is currently no agreement about which antiepileptic drug is most or least teratogenic; therefore, the drug
that works best for the patient should be used. Early
detection of neural tube defects by serum alpha-fetoprotein
screening, amniocentesis or ultrasonography is important. If
major abnormalities are present, the pregnancy can be

Topics: 23
Posts: 118

#17

terminated or an optimal management strategy can be


planned.
Switching to another drug is not usually recommended for the
patient taking anti-epileptic drugs during pregnancy.
According to the American Academy of Pediatrics (AAP) and
American Academy of Neurology (AAN), breastfeeding should
be encouraged in women with epilepsy. This is due to the
belief that the benefits of breastfeeding outweigh the risk of
exposure of the infant to antiepileptic drugs.
Ethics: Expectant teenage mothers are legally entitled to give
consent for procedures related to the management of their
pregnancies, including the performing of cesarean sections.
Adolescents do not need consent from their parents for
contraceptive services. Barrier methods should be used by all
sexually active adolescents even if an additional method of
contraception is being used.
Most states have implemented laws that allow for a physician
to provide certain types of medical care to adolescents
without parental consent. Elective abortions are typically
protected under these laws. Therefore, a teenaged girl would
have the authority to provide legal consent for an abortion.
When deciding ethical issues such as this, it is important to
focus on the principles of patient autonomy and beneficence.
Patients have the right to seek, accept, or refuse care. With
the mentally disabled, it may be impossible to obtain
informed consent, but at the least simple assent should be
strenuously pursued. If sterilization is not wanted, it should
not be performed against the patients will. Involuntary
sterilization violates a womans right to privacy, her
reproductive rights, and her bodily integrity.
Lesbian women often conceal their sexual orientation from
their primary care physicians. Recent developments and
legislations regarding same sex marriages have encouraged
more homosexual couples to "come forward" or reveal their
sexual orientation. It is important for primary care physicians
to be sensitive and knowledgeable to their special needs and
concerns.
Contraception: Levonorgestrel is the recommended method of
emergency contraception if used soon enough after an
unprotected sexual intercourse. It has maximal efficacy when

used within the first 12 hours after intercourse, good efficacy


within 48 hours, and appears to work up to 120 hours after
intercourse.
One controlled trial showed that levonorgestrel prevents 85%
of expected pregnancies (compared to 57% prevented by
estrogen/progesterone combination).
The copper intrauterine device is an effective emergency
contraception tool that can be used if a patient presents more
than 120 hours after unprotected intercourse.
The most effective reversible contraceptive methods are
injectable contraceptives (97 to 98% in 1 year) and oral
contraceptives (92%). Oral contraceptives (OC) are
contraindicated in patients with a history of thrombosis,
estrogen-dependant tumors, or liver disease, as well as those
who are heavy smokers or older than 35 years. They are also
not recommended for patients who are younger, on
anticonvulsant medication, have a heavy smoking history,
have migraines, or uncontrolled hypertension. Depot
medroxyprogesterone is ideal for women with heavy
menstrual loss, fibroids, a history of PID, or heavy smoking.
Implantable and injectable contraceptives, including
implantable levonorgestrel and depot medroxyprogesterone
acetate, have the lowest rate of pregnancy (does not exceed
2-3%) among nonpermanent methods of contraception.
"Norplant" consists of six capsules of levonorgestrel which are
placed subdermally, generally in the upper arm. It offers
contraceptive protection for about five years. The most
common complication is menorrhagia (prolonged vaginal
bleeding during each period), which occurs in about 28% of
the cases.
The absolute contraindications to the use of oral
contraceptives are the following:
1. history of thromboembolic event or stroke
2. active liver disease
3. history of estrogen dependent tumor
4. pregnancy
5. abnormal uterine bleeding
6. heavy smokers who are older than 35
7. hypertriglyceridemia
Relative contraindications are the following:
1. migraine headaches
2. poorly controlled hypertension

3. anticonvulsant drug therapy


Numerous commonly used antiseizure medications (e.g.,
phenytoin, carbamazepine, ethosuximide, phenobarbital,
topiramate) are known to decrease the efficacy of oral
contraceptives.
Alternative antiseizure medications that do not decrease oral
contraceptive efficacy include gabapentin and valproate.
Combination oral contraceptive pills (OCPs) are the most
reliable form of reversible contraception; however, the use of
these hormonal drugs may affect a person"s TBG, total T4,
and free T4 levels. The estrogen component of the
combination pill increases Thyroxine Binding Globulin (TBG)
levels, which may result in an increase in the total T4 levels
and decrease in free T4 levels. A person with an intact and
normal thyroid gland may compensate for these changes by
increasing thyroid hormone production, which normalizes the
free T4 levels. On the other hand, a hypothyroid patient
cannot compensate for these possible physiologic changes
since she is unable to produce sufficient amounts of thyroid
hormone and is dependent on levothyroxine therapy. The
dosage of levothyroxine may therefore be increased in
hypothyroid patients who are taking OCPs to compensate for
the mentioned physiologic responses. After starting
combination OCPs in well-controlled patients with
hypothyroidism, TSH levels should be checked in 12 weeks,
and the dose of levothyroxine should be adjusted accordingly.
Studies have shown that adolescents in a private practice
setting are very concerned about the prospect of gaining
weight secondary to oral contraceptive (OCP) use. However,
available data has demonstrated that OCPs do not cause an
increase in body weight or percent body fat. Occasionally
adolescents voice other concerns about OCP use, including
fears of developing blood clots, birth defects, and infertility,
but these concerns are more common in the lower
socioeconomic groups.
Chronic OCP can cause hypertension, discontinue it and
problem goes away.
SLE pts with increased risk of thrombosis (antiphospholipid
and nephritic synd) and pts with active renal involvemnt
should avoid OCP.
Pt with Migrain should avoid OCP due to increased risk for
Stroke. Breast cancer has NO asso with COP. Endometrial and

Ovarian cancers risk is decreased with use if OCP.


When you 1st start taking it it could cause irregular bleeding
and that is the mcc of incompliance. Tell the pt to keep taking
it and it will be OK.
The accidental usage of oral contraceptives during the first
trimester of pregnancy is not associated with an increased
risk of fetal malformation.

#18

drtanvir

Dec 22, 2008 - 7:46 AM

Forum Senior

Human papilloma virus (HPV): infection is a sexually


transmitted infection strongly linked with the development of
cervical intraepithelial neoplasia (CIN) and cervical cancer.
Early initiation of sexual activity, multiple new sexual
partners, and high-risk partners (partners with HPV infection),
are the main risk factors for the acquisition of HPV infection
and cervical cancer. Screening for cervical cancer by cytologic
examination/Pap smear is an effective way of detecting early
pre-invasive and invasive carcinoma.
According to the guidelines, screening for cervical
intraepithelial neoplasia (CIN) or cancer should be started
three years after the onset of sexual intercourse, or at the
age of 18 years (recently changed to 21 years) whichever is
earlier. This is due to the fact that high-grade cervical
cytologic abnormalities due to HPV usually do not occur until
three to five years after HPV exposure.
There are different approaches for the evaluation of ASCUS.
HPV DNA testing is the best way to evaluate atypical
squamous cells of unknown significance. In this method,
samples are collected both for cytology and HPV DNA, and if
cytology results are negative, the sample for HPV DNA is
discarded. If cytology results are positive, HPV DNA testing is
performed, and if this test is positive for high-risk HPV type,
an immediate colposcopy is performed. If the test is negative
for high-risk HPV type, the Pap smear is repeated after one
year.
High-grade squamous intraepithelial lesions include CIN II,
CIN III, moderate and severe dysplasia, and carcinoma in
situ. High-grade squamous intraepithelial lesions are more

Topics: 23
Posts: 118

likely to be progressive than low-grade lesions, and these


should always be treated with ablation or excision. Ablation
can be done using cryosurgery or laser, and excision can be
done using knife conization, laser conization or Loop
Electrosurgical Excision Procedure (LEEP). Loop
Electrosurgical Excision Procedure (LEEP) is the treatment of
choice for high-grade squamous intraepithelial lesion. LEEP is
preferred because of its low cost, accuracy, and easiness to
perform. It is a very successful procedure and can be
performed in an office setting.
Most of the low-grade squamous intraepithelial lesions (LSIL)
or low-grade cervical intraepithelial neoplasia (CIN I) regress
spontaneously; therefore, expectant management is preferred
for biopsy proven CIN 1 with satisfactory colposcopic
examination. A colposcopic examination is satisfactory when
an entire lesion and a transformation zone are visualized.
Expectant management includes repeat cytology at 6 and 12
months, or HPV DNA testing at 12 months. If there is
progression during the follow-up, or lesions are persistent
after one year, treatment is indicated.
When the colposcopic exam is unsatisfactory, the next step is
always excision as this allows for histological examination.
Even if colposcopic examination is satisfactory, treatment may
be indicated if the patient is anxious about her disease, or if
she seems to be non-compliant with follow-up. If the patient
opts for treatment, available modalities are either ablation or
excision. Ablation can be done with cryosurgery or laser, and
excision can be done with knife conization, laser conization, or
Loop Electrosurgical Excision Procedure (LEEP).
A HGSIL revealed on Pap smear indicates a 1-2% probability
of already having invasive cervical cancer and a 20%
probability of acquiring invasive cervical cancer if left
untreated. Immediate referral to colposcopy and endocervical
curettage is indicated. If colposcopy suggests HGSIL, a
diagnostic excisional procedure should be performed.
Human papilloma virus (HPV) infection has been strongly
linked with the development of cervical intraepithelial
neoplasia (CIN) and cervical cancer. Sexual intercourse,
especially with multiple new partners, is the main risk factor
for the acquisition of HPV infection. The risk of acquiring HPV
infection (and, hence, CIN/cervical cancer) is much lower in
lesbian women if they do not engage in sexual intercourse

with men. Lesbian women still need to undergo routine


screening for cervical cancer. Annual screening by
Papanicolaou smear is recommended for all women
approximately three years after the onset of vaginal
intercourse, or at the age of 21, whichever is earlier. The risk
of acquiring HPV infection (and CIN/cervical cancer) is lower
in lesbian women if they do not engage in sexual intercourse
with men. The risk of cervical neoplasia is highest in lesbian
women who have had sex with more than one male sexual
partner, have an early age at first coitus with men, have been
infected with HPV, and have been treated for an abnormal
cervical cytology test in the past. The risk is also increased
with cigarette smoking.
Bleeding is the most common short-term complication after
cervical conization (cone biopsy), both cold knife conization
and LEEP. The causes of bleeding include inadequate
intraoperative hemostasis and vasodilatation after the effect
of the local vasoconstrictor wears off. Usually, the bleeding
can be easily controlled by conservative measures. Surgical
hemostasis is rarely required.
All women at high risk of HPV infection and a normal Pap
smear should have annual Pap smear tests.
Cervical stenosis: Surgical treatment of cervical intraepithelial
neoplasia before conception increases the incidence of
cervical incompetence and preterm birth 200-300%. It is
thought that the risk is proportional to the amount of tissue
removed during the procedure. Frequent cervical
examinations in the second trimester will aid in determining if
either cerclage is necessary in the short term or cesarean
section is necessary in the long term.

HIV: All HIV-positive adults are normally counseled to receive


HAART (highly active antiretroviral therapy), which is a
combination of two nucleoside analogue reverse transcriptase
inhibitors and one protease inhibitor or one non-nucleoside
reverse transcriptase inhibitor. An HIV-positive woman who is
already receiving HAART should be counseled to continue her
treatment regimen if her pregnancy is identified after the first
trimester. (Zidovudine in particular is considered an essential
component of the treatment regimen as it has been

demonstrated to significantly decrease the vertical


transmission of HIV.) For those HIV-positive women whose
pregnancies are identified early on in the first trimester, an
acceptable alternative is to discontinue HAART for the
remainder of the first trimester and then resume it thereafter.
Ultimately, the preferred mode of delivery in all HIV-positive
women is elective cesarean section.
Efavirenz and delavirdine are thought to have some
teratogenic qualities and are best avoided during pregnancy if
other options are available.
Studies have shown that HIV can be transmitted through
breast milk. Worldwide, it is estimated that one of every eight
babies born to HIV-positive women will acquire the virus
through breastfeeding. For this reason, HIV-positive women in
the United States and other industrialized countries who have
ready access to commercially prepared formula are advised to
feed their infants with formula instead of breast milk.
Most antiretroviral drugs are passed through breast milk in
significant quantities, which is thought to decrease HIV
transmission by indirectly providing the drugs to the infant.
This understanding accompanied by the concern that
formula is often too difficult or expensive to obtain in
developing countries is the justification for suggesting
that HIV-positive women in developing countries be given
HAART while breastfeeding their children. However, the
recommendation in the United States remains that HIVpositive women feed their infants with formula instead of
breast milk, regardless of therapy given to the mother or
infant.
Condyloma acuminate: This is the dermatologic manifestation
of an infection with the human papilloma virus, with over
90% of such condylomas arising from HPV subtypes 6 and 11
specifically. HPV is primarily transmitted through sexual
contact, and the areas affected include the penis, vulva,
vagina, cervix, perineum, and perianal region. Less
frequently, HPV may be found in the oropharynx, larynx, or
trachea secondary to oral-genital contact or secondary to
vertical transmission from mother to infant during childbirth.
One relatively common benign laryngeal tumor in children,
recurrent respiratory papillomatosis, is caused by the
acquisition of HPV during passage through the vaginal canal.

However, since HPV is thought to be contracted by the infant


in less than 1% of all childbirths to women who have
condylomas, no intervention is usually undertaken
Pelvic inflammatory disease (PID): The risk factors for PID
include the following:
1. Oral contraception
2. No barrier contraception
3. Multiple sexual partners
4. Age < 35 years
5. History of previous episodes
6. African-American ethnicity
Studies have shown that among all the PID risk factors,
having multiple sexual partners is the one associated with the
highest increase in risk for occurrence of PID (4.6 to 20 fold
increase in risk).
The presence of fever, chills, bandemia, leukocytosis, as well
as nausea and vomiting, indicates that the infection is severe.
PID can be a serious condition that needs to be treated
immediately. Treatment of severe PID is aimed at obtaining
high blood concentrations of the antibiotics as soon as
possible. For this reason, all therapy should be intravenous.
Combinations that can be used are: cefoxitin + doxycycline or
ceftriaxone + doxycycline. If a tubo-ovarian abscess is
diagnosed, surgical evaluation is immediately needed.
Tubo-varian abscess Is seen in 10% of pt w PID. Admit the pt,
Broad spectrum antibiotics should be started immediately
AFTER taking cultures. Gentamycin+Clindamycin+Ampicillin.
In absence of obvious response w/i 48 hours , drainage
should be considered. If there is doubt re Dx we do
Laparoscopy. If rupture we need to do exploratory
Laparotomy.
Vaginitis, Candida Its not an STD. It may appear in presence
of risk factors like DM, OCP, pregnancy and immunosupresive
therapy. It can also trigger by Alkaline pH of the vagina during
menses that is favorable for growth of Candida. Symptoms
are vaginal and vulvar pruritis, burning and dysparunia.
Discharge has cottage cheese appearance. Its NOT
malodorous and pH is 4-4.5. Dx is by KOH wet mount
preparation and visualization os pseudohypha and spores. Tx
is Imidazole cream or 10 day oral Ketoconazole (especially if

recurrent). Fluconazole is the new drug with fewer SE. Give 2


doses, a week apart. Most recurrences are due to noncompliance. If that is OK, then suspect anoterh oraganism
and culture on Sabourausd. 50% of reccurence is due to other
organism like like T.Glabrata and Candida tropicalis. In
addition, 35% of pt with recurrent yeast infection and who
have negative cultures, have HPV on biopsy.
DD: Trichomonas (Metro)
Gardenella Vaginitis (Metro).
Mittelschmerz: Its a mid-cycle (LMP was 2 weeks ago)
abdominal pain that typically presents in young females. If
there is no fever its not PID.
Vaginismus: Involuntary spasm of the perineal musculature
that interferes with sextual intercourse.
Unlike Dysparunia, Vaginismus is not due to any medical
cause. In ALL cases its due to ignorant of women's anatomy,
ahuge apprehention about penetration, and an uncontrolable
fear of pain. Tx includes relaxation, Kegel excercise (to relax
vaginal muscle), and graduation dialation (penetration).
Chlamydia trachomatis: Genitourinary infection caused by
Chlamydia trachomatis is extremely common, and results in
insidious and often chronic unrecognized disease. The large
majority of women (up to 70 percent) are asymptomatic. It is
a major cause of infertility, pelvic inflammatory disease (PID),
and ectopic pregnancy in women. Some of the risk factors
associated with the development of Chlamydia trachomatis
infection include younger age with multiple sexual partners,
inconsistent use of barrier contraceptives, and a history of
prior sexually transmitted disease (STD). The Centers for
Disease Control recommend that all pregnant women be
screened for Chlamydia at the first prenatal visit. Women
under age 25 and those at increased risk for chlamydial
infection should have repeat testing in the third trimester.
Chlamydia endometritis during pregnancy can lead to
chorioamnionitis and premature delivery of the fetus.
Untreated infection during pregnancy can also lead to
conjunctivitis (ophthalmia neonatorum) and pneumonia in the
newborn baby. Chlamydia infection is a sexually transmitted
disease. Pregnant women under age 25 and those at
increased risk for chlamydial infection should have repeat

testing in the third trimester.


Chlamydia infections are susceptible to tetracyclines,
macrolides, and fluoroquinolones. Azithromycin (1 g PO as a
single dose) and doxycycline (100 mg PO BID for 7 days) are
the two most commonly used and recommended treatment
regimens in the general population. Some of the other less
frequently used alternative regimens include seven days of an
erythromycin base (500 mg PO QID), erythromycin
ethylsuccinate (800 mg PO), ofloxacin (300 mg PO BID), or
levofloxacin (500 mg PO QD). Presumptive treatment of the
partners is crucial in the management of these patients to
prevent recurrence/reinfection. Doxycycline, the
fluoroquinolones, and erythromycin estolate are
contraindicated for use in pregnant women. The
recommended regimens during pregnancy include
erythromycin base (500 mg PO QID for 7 days) and
amoxicillin (500 mg PO TID for 7 days). Azithromycin (1 gm
as a single dose) is an alternative, though it has not been well
tested in pregnancy.
Lymphoganuloma Venereum Its a STD caused by
C.Trochomatis. Serotype L1L2L3. Initially there is headache
and fever, then a papul apears that turns in to an ulcer
typicaly in vulvovaginal region. Ulcer is painless and disease
may go un noticed unti inguinal adenitis develops a month
after. If untreated at this stage LGV becomes chronic causing
ulceration, PROCTATITIS, rectal stricture rectovaginal fistulas
and elephantiasis. Tx is Doxycycline or erythromycin .
DD:Granuloma Inguinale is caused by Donovania
granulomatis, Unlike LGV the ulcer and lymphadenopahty
present simultaneously. Also the ulcer here has irregular
borders and beefy red granular base. Recommended Tx
include azithromycin, doxycycline, erythromycin.
Trichomoniasis: Trichomoniasis is a sexually transmitted
infection caused by the protozoa Trichomonas vaginalis. The
infection may be entirely asymptomatic or may be
characterized by a copious frothy gray-green vaginal
discharge, fishy odor, vulvar and vaginal pruritus, or dysuria.
Treatment is indicated for all women diagnosed with
trichomoniasis and their sexual partners, and metronidazole is
considered the treatment of choice. Metronidazole is known to

enter breast milk in significant concentrations, but few


adverse effects have been reported in infants. Therefore most
physicians recommend that lactating women be treated with
metronidazole 2g PO for 1 dose, and that breastfeeding be
discontinued for 12-24 hours only.
Vulvovaginitis: is an extremely common condition in the
sexually active age group. The etiology of symptoms includes
infection of the vulva, vagina, and cervix, and the
noninfectious causes are chemicals and irritants, hormonal
disorders, and, rarely, some systemic disorders. Common
infectious causes are vulvovaginal candidiasis, bacterial
vaginosis, and trichomoniasis. Most of the symptoms of
vaginitis are nonspecific, and self-diagnosis and treatment by
the patient is unreliable. Even diagnosis by history and
physical examination by a physician is not reliable and should
always be confirmed in the office by microscopy. This is
important to avoid frequent misdiagnosis and inappropriate
treatment. Most common techniques used are saline and KOH
preparation for microscopy. The vaginal secretions have
characteristic pseudohyphae in candidiasis. Bacterial vaginosis
is confirmed by the presence of clue cells in the smear. Motile
trichomonads and an abundance of polymorphonuclear cells
are found in patients with trichomoniasis.
Pelvic cystic mass: A pregnant woman with a pelvic cystic
mass needs to be observed carefully. If the mass is larger
than 5 cm in size and persists beyond the first trimester or
shows no regression in subsequent visits, surgical
intervention is indicated. Removal of the cyst is recommended
during the second trimester to avoid acute complications that
lead to preterm delivery or abnormal labor.
Ascites: Abdominal paracentesis and ascitic fluid analysis is
the most reliable way to differentiate between the different
causes of ascites. The most common cause of ascites in the
United States is hepatic cirrhosis. Some of the other causes
are cancer, congestive heart failure, peritoneal tuberculosis,
nephrotic syndrome, and pancreatic disease.
The differential diagnosis of ascites can be narrowed down by
calculating the serum to ascitic fluid albumin gradient (SAAG).
This is done by subtracting the ascitic fluid albumin value

from the serum albumin value. SAAG value is useful in


identifying the presence or absence of portal hypertension.
SAAG value of > or = 1.1 g/dL indicates the presence of
portal hypertension. Clinical conditions associated with high
SAAG (> 1.1 g/dL), include cirrhosis, congestive heart failure
and alcoholic hepatitis. Conditions associated with low (< 1.1
g/dL) albumin gradient; include peritoneal carcinomatosis,
peritoneal tuberculosis, nephrotic syndrome, pancreatitis, and
serositis.

#19

drtanvir

Dec 22, 2008 - 7:47 AM

Forum Senior

Premenstrual syndrome (PMS): Abdominal bloating,


headache, fatigue, weight gain, anxiety, and decreased libido
7 to 10-days before the start of each menstrual cycle. The
presence of symptoms such as this patient is presenting, after
menstrual cycle begins or after ovulation, points out to the
diagnosis of premenstrual syndrome (PMS). Symptoms
usually disappear near menses and
the patient is symptom free until her next ovulation.
Psychological symptoms may include anxiety, mood swings,
difficulty concentrating, decreased libido and irritability.
PMS has to be differentiated primarily from major depression
because there is an association between puerperal
depression, menopausal depression and PMS. A menstrual
diary for at least 3 cycles is a useful aid for confirming the
diagnosis in suspected cases; PMS is confirmed when one or
several particular symptoms occur repeatedly at the same
time of the several cycles.

Topics: 23
Posts: 118

Once the diagnosis is confirmed, treatment depends on the


patient's complaints. There is no universally accepted
treatment. Reduction of caffeine intake may reduce breast
symptoms. Exercise program may be effective in improving
the general well being. Recently, selective serotonin reuptake
inhibitors have been shown to be beneficial in double blind
controlled trials and now the drug of choice for mood decline.

Amenorrhea: Amenorrhea is thought to occur in female


athletes when there is a relative caloric deficiency secondary
to inadequate nutritional intake as compared to the amount of
energy expended. Women athletes with this condition have
been shown to have decreased levels of luteinizing hormone
(LH) and gonadotropin-releasing hormone (GnRH), resulting
in an estrogen deficiency. These amenorrheic women are
therefore at increased risk for all conditions associated with
estrogen deficiency, including infertility, vaginal atrophy,
breast atrophy, and osteopenia.
Females who maintain a lower weight or body mass index
(BMI) due to the sport or activity that they regularly engage
in (e.g. ballet dancers, gymnasts, and runners) may become
hypoestrogenic and present with exercise-induced
amenorrhea. They are at special risk of developing
osteopenia, and even osteoporosis. Spontaneous fractures
have been reported in these types of patients, with
osteoporosis or osteopenia confirmed through bone
densitometry. Treatment consists of improving caloric intake;
if this is not possible, patients are started on hormonal
replacement with oral contraceptives and supplementation
with calcium and vitamin D.
The most common cause of secondary amenorrhea is
pregnancy. Drugs that inhibit or induce the cytochrome P450
system can reduce oral contraceptive efficacy.
Infertility: The first step in WOMEN is to check Basal Body
Temperature and mid luteal PROGESTERONE. The ovulatory
factor involves defects in the hypo-thalamic pituitary ovarian
axis, and related infertility maybe due to impairment of
follicular maturation ovulation or endometrial development.
BBT assess the DURATION luteal finction and MLP asseses
LEVEL of lutal function. Endometrial biposy is done to confirm
luteal phase defect. rather than initial evaluation.
MALES: According to the recent studies, male factors account
for 20-30% of the infertility causes. Semen analysis is
therefore performed early in the evaluation of the infertile
couple. It is usually the initial screening test, and it evaluates
sperm concentration, motility, and morphology. Identification
of azoospermia and severe oligospermia are likely causes of
infertility. Although cutoff values for semen analysis exist,
there is a broad overlap in the values of the semen

measurements in fertile and infertile samples; therefore,


borderline results should be interpreted with caution. If sperm
analysis is normal then an endocrine hormonal evaluation is
carried out. It includes: 1-TFT (since increaed TSH inhibits
GnRH and then decrease FSH. 2-Testosterone levels to
indicate the presene or not of Gonadism. 3-Gonadotropin to
determine whether hypogonadism is central or testicular and
4-Prolactin lelevs.
Causes of infertility in femlaes falls in 4 factors: 1-Peritoneal
factor. 2-Ovulatory. 3-Cervical. 4-tubo-uterine. Peritoneal is
the mc type and includes Endometriosis and peritoneal
adhesions. Laparoscopy is the procedure of choice. for dx and
tx. Mild forms of endometriosis usually respond to meds like
GnRH agonists, Danazol and Medroxyprogesterone. 2Ovulatory factor involves hypothalamus-pit-ovary axis. and
infertility might be due to impairment of follicular maturation,
ovulation,or endometrial development. ovulatory abnormality
may initially be screened by Basal body temp and midluteal
phase level of progesterone, the former asseses DURAtion
and later LEVEL of luteal function. If luteal phase shows low
progesterone, hence infertility, then tx is suppository
progesterone deposition. 3-Tubo0uterine is seldom a cause. It
onvolves Fibroids, endometrial polyps, tubal occlusion(2ary to
IUD or endometriosis). Investigation is ainlt
hysterosalpingography or laparoscopy. 4-Cervial involves
cervial structure abnormalities and abnormal mucus
production. In 5-10% infertility remains unidentified.
Intrauterine insemination is the tx.
Clomiphene Citrate is an antiesterogen that acts by
competitively inhibiting esterogen receptors at hypothalamus, thus inhibiting the negative feed back esterogen
has on GnRH production and consequesntly increasing LH &
FSH secretion and improving ovulation. Along with HCG and
HMC its indicated for chronic anovulaation. Side effects
include large ovaries, hot flashes, abdominal bloating, breast
discomfort and abnormal uterine bleeding.
A retroverted uterus is a common condition found in 11% of
the female population. If this condition is secondary to chronic
PID or endometriosis, it can be associated with infertility. It is
also related to dyspareunia, low back pain, and
dysmenorrhea. If it is not spontaneously corrected during
pregnancy before the second trimester, it can lead to uterus

incarceration and be life-threatening. It is not an etiologic


cause of abortion.
Chances of infertility in a female with cystic fibrosis are 20%
percent. This increased risk is due to secondary amenorrhea
caused by malnutrition, and is also due to thick, tenacious
cervical mucus. Chances of infertility in a male with cystic
fibrosis are 95%. Spermatogenesis is normal in a patient with
cystic fibrosis, but sperm transport is impaired because of
impaired development of the Wolffian duct.
A defect in the luteal phase: A defect in the luteal phase
refers to a relatively low progesterone secretion by the corpus
luteum. Progesterone is important for embryo implantation
and maintenance of pregnancy, so a defect in its secretion
may cause infertility even if ovulation takes place. The defect
may affect the level or duration of production of
progesterone. Luteal phase defect is suggested by short
cycles, history of spontaneous abortion, abnormal basal body
temperature (BBT) or low levels of midluteal progesterone.
The diagnosis is confirmed by endometrial biopsy, which
demonstrates a lag in endometrial maturation of 2 days or
more as compared with normal states.
Luteal phase defect is treated with progesterone vaginal
suppositories. Clomiphene citrate or human menopausal
gonadotropin (hMG) may be used if progesterone
suppositories give no results; they increase serum FSH level
and thus correct follicular maturation and oocyte expulsion
As mentioned above, luteal phase defect refers to a deficiency
in progesterone production; therefore, combined oral
contraceptive pills are not appropriate.
Danazol is an androgen derivative that has a gonadotropin
inhibitory effect. It is indicated in endometriosis, fibroids and
fibrocystic breast disease.
Polycystic ovarian syndrome (Stein-Leventhal syndrome):
infertility, menstrual irregularity, obesity, hirsutism, and
increased ovarian volume measured by ultrasonography.
Anovulation is the principal cause of infertility in patients with
polycystic ovarian syndrome. Interestingly, several studies
have demonstrated that weight reduction in overweight or
obese patients with this syndrome is associated with
restoration of ovulation, decrease in androgen production,

and pregnancy. Furthermore, the data from these studies


indicate that even a moderate reduction in weight may
restore fertility.
Insulin-resistant hyperinsulinemia is a common finding in
patients with PCOD.
PCOD is an important risk factor for development of DM-2 in
women. About 10% of diabetes in premenstural women is
POS related. Thus a glucose tolerance test is needed in all
POS pts. A two hour test with > 140 is Dx and needs furthur
action. Life style modification and Metformin is then indicated.
Metformin helps in following ways: 1-prevents DM, 2-Causes
anorexia and prevents obesity, 3-corrects hirsutism mildly, 4menstural irregularity and infertility
The treatment of PCOS is directed primarily at the problems
of hirsutism, menstrual irregularity, and infertility. Treatment
modalities for hirsutism include ovarian and adrenal
suppression, antiandrogen therapy, and local depilatory
measures. Oral contraceptives are a simple and relatively safe
method of ovarian suppression and will often normalize the
menstrual cycle. The estrogen component in oral
contraceptives increases the sex hormone binding globulin
with a resultant decrease in free testosterone. This reduces
the symptoms of excessive hair growth and acne.
Clomiphene citrate is the drug most commonly used to induce
ovulation in patients with polycystic ovary syndrome (PCOS).
This condition should be suspected in any pt who has
mentrural irregularities, and evidence of
Hyperandrogenism(facial hair). Presence f these establishes
the dx.50% of pts are obeseand are at risk of DM-II. The next
step is to do Oral glulcose tolerance test
Characterized by unbalanced Estrogen secretion resulting in
Endometrial Hyperplasia.
Ovarian HyperStimulation Synd Major complications of
ovulation induction are multiple gestation and OHSS. OHSS
occurs in 1-3% of pts. It develops 5-7 days after
administrationof hMG or hcg and refers to acute increase in
vascular permeability of enlarged ovaries. Symptoms include
abdominal pain, distension, nasea, vomiting diarrhea, weight
gain,hypotension and pleural effusion. Ovaries are large in
US. OHSS maybe complicated with oravian torsion, ovarian
rupture, renal insufficiency. Symptoms resolve in 1-2 weeks,

but may persist longer is pregnancy is successful. Tx is


supportive.
Androgen insensitivity syndrome:The combination of primary
amenorrhea, bilateral inguinal masses (one removed earlier
and one palpable in this case), and breast development
without pubic or axillary hair is strongly suggestive of
androgen insensitivity syndrome. This condition is related to a
mutation of the androgen receptor (AR) gene, making
peripheral tissues unresponsive to androgens that are
typically available in normal concentrations in these patients.
Although the genotype is 46, XY, a female phenotype
develops. Breast development is present, because
testosterone is converted to estrogen; however, there is no
pubic or axillary hair. No mullerian structure is present
(uterus, fallopian tube), and the vagina ends with a blind
pouch.
Primary amenorrhea is usually due to chromosomal
abnormalities and physiologic delay of puberty (45% and 20%
of causes, respectively). In both cases, development of
secondary sexual characteristics is delayed. Imperforate
hymen presents as cyclic, pelvic or abdominal pain with
primary amenorrhea. Sometimes, a small suprapubic mass
(the uterus containing retained menstrual blood) can be
palpated. Perineal exam will reveal a bulging, bluish
membrane between the labia. This bluish or violaceous
discoloration is due to blood sequestration behind the
imperforate hymen, and is also known as hematocolpos.
Tetsticular feminization defect or absence of androgen
receptor results in feminine phenotype with 46XY genotype.
MIF is produced by gonads, so urtus and vagina are absent.
Breat develop because peripheral production of estrogen ,
whereas axillary hair and pubic hair does not. Tx is testicular
resection at puberty and creation of aneo vagina. Pt prestns
with amenorrhea, developed breasts, absent pubic and axllary
hair , absent internal reproductive organs and a 46XY
karyotype.
Turner syndrome: Ovarian function and morphology is highly
variable in patients with Turner syndrome. Most patients do
not undergo puberty, and present with primary amenorrhea.

Some patients may develop normally and undergo


spontaneous menarche, but secondary amenorrhea may
occur after some time. Such patients may become pregnant
without medical assistance before developing secondary
amenorrhea; however, most women are infertile. In one
retrospective study of 522 patients, three women became
pregnant spontaneously.
Summary: Pregnancy is almost impossible, but there still
remains a very small chance, in patients with Turner
syndrome.

Endometriosis: is a progressive disease typically diagnosed in


women aged 25 to 29 years, and is thought to affect 5-10%
of all women. It is characterized by the presence of
endometrial tissue in the ovaries, fallopian tubes, or other
abnormal sites, and commonly causes significant pelvic pain
and infertility. Typically pt prestns with Dysmenorrhea,
Dysparunea(when endometriomas is in cule-de-sac),
Dyschezia(Pain on defecation), hematochezia, hematuira, and
pre post menstrual spotting.
Endometriosis should be considered a strong likelihood in
women who experience dysmenorrhea after years of painless
menstruation. Diagnosis is difficult and is best done with
direct visualization of the implants by laparoscopy or, less
commonly, laparotomy. Laparoscopy is GOLD standard which
shows powder burns. The hemorrhage of endometriomas into
the ovaries results in formation of cystic cavity filled with
blood with dark color, hence the name 'Chocolate cyst'. To
complicate matters, there may be little correlation between
the clinical manifestations and the actual extent of
endometrial implants.
The most frequent sites of involvement are the ovaries, culde-sac, broad ligaments, uterosacral ligaments, uterus,
fallopian tubes, and sigmoid colon. Endometriosis may also
involve the appendix, cecum, and colon, which may lead to
intestinal obstruction.
Endometriosis can be treated medically or surgically. The first
line of tx is OCP. They cause a state of pseudopregnancy and
causing an "exhaustion atrohpy" of the endometriomas. If
OCP fails or not tolerated then we give Danazole, its an
androgen deivative that causes Pseudomenopause state. SE

are acne,hirsutism deep voice.GnRh agonists have an


inhibitory action of LH & FSHwhen given continuslywhich
produces temporary castration. Its also a 2nd line of choice.
Surgical treatment involves removal of lesions through bipolar
coagulation or laser vaporization, while severe cases may
require hysterectomy and bilateral salpingo-oophorectomy.
In patients with severe or unresponsive symptoms (including
infertility), the standard of care requires that confirmatory
laparoscopy be performed to definitively establish the
diagnosis of endometriosis before instituting treatment.
DDX:Vaginismus, use Vaginal dialators.
Pain disorder, pain in one or more anatomical sitetxis pain
managment training.
Somatization disorder, tx is follow up visitsregularly
scheduled.
Endometriosis is the location of tissue outside uterine cavity
so hysterosalpingogram cant see it.
Dysfunctional uterine bleeding (DUB): Heavy unremitting
endometrial hemorrhage through the menarche and
perimenopuase requires high dose conjugated estrogens to
suppress bleeding, and to ensure cardiovascular stability.
Once cardiovascular stability is achieved, D&C should be
performed.
The most common cause of dysfunctional uterine bleeding
(DUB) in adolescent women is anovulation. So, endometrial
biopsy is not required in these patients. Once the bleeding is
stopped, advise the patient to: take 25 mg of oral conjugated
estrogen daily for 25 days; add 10 mg of
medroxypreogesterone for the last 10 to 15 days of
treatment; then allow 5-7 days for withdrawal bleeding. All
this in order is to mimic the menstrual cycle.
In which pts with DUB do you perform endometrail biopsy to
rule out endometrial carcinoma? When a pt is >35, obese, DM
or has chronic HT.

drtanvir
Forum Senior

Dec 22, 2008 - 7:47 AM

#20

Menopause: Most females rapidly lose considerable bone


mass following menopause. Lifestyle modification, including

Topics: 23
Posts: 118

weightbearing exercises and optimum calcium and vitamin D


supplementation, prevents postmenopausal bone mass loss
and possibly reduces the risk of fragility fractures. The
National Academy of Science recommends daily
supplementation of elemental calcium (1200 mg) and vitamin
D (400 to 800 international units) in women after 50 years of
age. This can be easily achieved by taking one multivitamin
tablet (which contains 400 international units of vitamin D)
with two tablets of calcium/vitamin D (containing 600 mg of
elemental calcium and 200 international units of vitamin D)
everyday.
Short-term use of low-dose estrogen for menopausal
symptoms does not appear to be harmful; however, hormone
replacement therapy fell out of favor following a recently
published study of the womens health initiative because
of increased, undesirable cardiovascular and other side
effects.
Exercise with weightbearing leads to improvement in bone
mass, owever, exercise should be individualized in patients
with significant osteoporosis, since they have a higher risk of
fracture. In such patients, stressful exercises should be
avoided as much as possible.
Recently published womens health initiative studies show
that older postmenopausal women on hormone replacement
therapy are at an increased risk for myocardial infarction,
deep venous thrombosis, strokes, and breast cancer. The
findings of this study do not apply to younger patients with
premature ovarian failure, such as the patient described in
this vignette. Hormone replacement therapy in the form of
conjugated equine estrogen and medroxyprogesterone with
careful monitoring can be safely used in younger patients
without any excessive cardiovascular risk.
The use of bisphosphonates in older, postmenopausal women
has been extensively studied, and has been shown to be very
useful in the treatment of low bone density. On the other
hand, bisphosphonates are not very well studied in younger
patients with low bone density. The long-term effects of
bisphosphonates such as alendronate or etidronate are
unknown in younger patients. Furthermore, although
risedronate (bisphosphonates) may improve bone mineral
density in this patient, it will not improve her hypoestrogenic
symptoms; therefore, it is not the preferred drug regimen.

Premature ovarian failure: Premature ovarian failure refers to


a failure of estrogen production by the ovaries that occur in
women less than 35 years. It may be secondary to an
increased atresia or a reduced number of primordial follicles.
It is most commonly idiopathic but may also be due to
mumps, oophoritis, irradiation or chemotherapy. It can be
associated with disorders such as Hashimoto's thyroiditis,
Addison's disease, type I diabetes mellitus and pernicious
anemia, which supports the hypothesis of an autoimmune
origin. Diagnosis is confirmed by increased FSH serum levels
and in patients less than 30 years. A chromosomal analysis
becomes necessary in order to rule out the presence of a Y
chromosome.
Since patients with premature ovarian failure lack viable
oocytes, measures of ovulation induction, such as clomiphene
citrate, hCG and hMG, will be useless, and the only option
such patients have to restore fertility is egg or embryo
donation.
Ovarian failure, chemo induced chemotherapy causes failure
of follicular cells of the ovary resulting in decreased
production of estrogen and inhibin. This result in loss of
feedback inhibition of estrogen on FSH and LH , causing their
high levels. Inhibin causes feedback inhibition of FSH only, so
in the absence of inhibin, FSH levels are higher than LH which
is pathognomonic of ovarian failure. Clinically, pts have
Amenorrhea, hot flashes. They might develop anxiety,
depression and irritability. Loss of estrogen results in atrophy
of breast, vagina, myometrium.
Postmenopausal bleeding: The most common cause of
postmenopausal bleeding is atrophic vaginitis (50-60%).
However, one should always rule out the most serious
conditions, such as endometrial carcinoma (less than 10%) by
obtaining endometrial biopsy.
Atrophic vaginitis is characterized by vaginal dryness,
burning, dyspareunia, reduced vaginal secretions, and vulvar
pruritus. Occasionally, urinary symptoms such as dysuria,
hematuria, and discomfort are also seen. Pelvic examination
typically reveals loss of labial fullness, pallor of the vaginal
epithelium, and decreased vaginal secretions. The condition
commonly occurs secondary to declining estrogen levels in

postmenopausal women. In nonmenopausal women, estrogen


production can be countered by radiation therapy,
chemotherapy, immunologic disorders, oophorectomy, and
lactation. Atrophic vaginitis is treated with hormonal
replacement therapy, transvaginal estrogen replacement
(e.g., creams, hormone-releasing rings), or lubricants.
DD: Vulvar cancer most often occurs in women aged 65-75
and is characterized by a long history of pruritus. Vulvar
bleeding, discharge, dysuria, and pain are reported less often.
A raised vulvar lump or mass is apparent on pelvic
examination, and may be fleshy, ulcerated, leukoplakic, or
warty in appearance.
Cervical cancer most often occurs in women aged 40-60.
Women at increased risk for developing cervical cancer
include smokers and those with a history of sexually
transmitted diseases, human papillomavirus infection, low
socioeconomic status, two or more lifetime sexual partners, or
immunosuppression (e.g., AIDS). Cervical cancer is
responsible for less than 1% cases of postmenopausal
bleeding.
Breast mass: Most of the palpable breast masses in young
patients (less than 35 years of age) are due to benign
etiology, most commonly benign fibrocystic disease or
fibroadenoma of the breast. The symptoms of fibrocystic
disease may get worse during pregnancy due to the hormonal
changes in the body. Despite these facts, any complaints of a
dominant mass in the breast should be thoroughly evaluated
to exclude malignancy. A dominant breast mass characterized
by a hard, single, and immobile mass with irregular borders
should raise the suspicion for malignancy. A cystic breast
mass should be evaluated with fine needle aspiration biopsy
to obtain tissue or fluid for a more specific diagnosis.The goal
of aspiration is complete drainage of the cyst and collapse of
the cyst wall. Clear cyst fluid may be discarded, with a follow
up exam in 4-6 weeks. If the mass persists at that time it
may be re-aspirated and further workup may be warranted.
Any blood-tinged fluid obtained should be sent for cytologic
analysis. If the fluid is non-bloody, the patient should be
reassured and reexamined in four to six weeks to check for
any signs of recurrence.
Excisional biopsy is invasive and does not need to be the first

step in a breast cyst evaluation. If the mass persists,


definitive evaluation with a biopsy would be an appropriate
next step.
Fibroadenoma: 1x1 cm firm rubbery freely mobile round mass
in a 35 yo women w/o axillary nodes palpable. Best initial
step is Mamogram.
Breast Carcinoma: Inflamatory beast cancer: Erythema and
edema of non lactating breast could be due to locally
advanced cancer, biopsy should be done first to rule out that
dx. Tx is 2-3 weeks of combination chemotherapy to shrink
the tumor allowing sybsequent extended resection.
Metastatic Breast Cancer has a poor prognosis. with little
chance of cure. Its importnat to choose Local (Surgery) vs
Systemic (Systemic Chemotherapy) tx in pt with metastatic
breast cancer. Tumor burden, based on TNM staging, is
considered the single most important prognostic consideration
in treating pts with breast cancer. ER+ and PR+ are good
prognostic factors. Over expression of Her2/Neu oncogene is
worst progosis.
Breast cancer is the leading cause of metastatic skin disease
in women. These lesions are erythematous that present as
erosions covered by necrotic skin. Tx is palliative radiation
therapy with aggressive wound care.
Two proibitions when having breast cancer in pregnants: No
chemo in 1st trimester, and no Radiotherapy anytime in
pregnancy. Also Lumpectomy is not a good choice for 1st
trimester cuase it needs Radiation afterwards. So the only Tx
for 1st trimester is Modified Radical Mastectomy.
Nipple discharge: Nipple discharge is a relatively common
problem, and is due to a benign cause in most cases. A
detailed history and physical examination can essentially
determine the differential diagnoses and necessary diagnostic
work-up. It is important to determine whether the discharge
is spontaneous or provoked, and whether it is unilateral or
bilateral. In general, patients with a malignant cause have a
spontaneous, unilateral and guaiac positive
or grossly bloody discharge. In contrast, patients with an
endocrine, medication-induced or other physiologic causes
tend to have a bilateral nipple discharge.

All patients with a unilateral spontaneous nipple discharge


should be screened for breast cancer with a mammogram.
Patients with evidence of a mass or suspicious finding on
mammogram should have further evaluation such as fine
needle aspiration biopsy or open breast biopsy. The discharge
should also be examined for the presence of occult or gross
blood; positive results are then sent for cytologic analysis.
Intraductal Papiloma: A benign tumor of lactiferous ducts.
Clinically manifest as serous bloody discharge. Mamo wont
show it, too small. Resection has to be done to relief pain
guided by galactogram.
Lactation suppression: Tight fitting bra and ice packs.
Bromocriptine is no longer used.
Mastitis Infection of breast with S. Aureus. It must be ddx
with Breast Engorgement (heavy,tender,firm and warm
breasts bilaterallyin women who not nursing. Its manged with
tight fiting bras, anagesics and ice pack, breast feeding
should be resumed). Mastitis is tx with oral Dicloxacillin.
Breast feeding should be suspended but milk has to be
pumped until infection clears. If it onvolves abscess, incision
and drainge is required.
Raloxifene Is a Selective Estrogen Receptor Modulator (SERM)
used for prevention of osteoporosis. Unlike Estrogen, it
doesnt increase risk of Endometrial cancer. It decreases
risk of Breast cancer. It has noeffect on Ovarian cancer. Ts
most important SE is it increases the risk of Pulmonary
Thromboembolism and is CI in pt with a hx of DVT. It may
also worsen hot flashes and vaginal dryness.
Tamoxifen: An antiestrogen drug used for breast cancer.
When used as adjuvent therapy for early stage disease it
reduces the risk of recurrence of original cancer and new
cancer in other breast. However it increases the risk of 2
types of cancer, 1-endometrial (lining of Uterus) and Uterine
Sarcoma.
It reduces the risk of breast cancer in those who are at
increased risk for developing breast cancer, ITS PROVEN. SEE
RALOXIFEN.

It increased the risk for endometrial cancer by 1% and ONLY


in postmenopasusal women. It decreased risk of breast
cancer, so overall it reduces mortality rate. It also decreases
risk facto in the opposite breast. It protects against
osteoporosis. However it does cause hot flashes and vaginal
dryness due to its antiesterogenic effect. Remember its mixed
agonist-antagonist on estrogen receptors.

Maternal substance abuse 1-Herion abuse: Newborn will show


tremors, increasd weakness, frequent loose stool, high pich
cry, fist sucking, poor feeding and tachypnea, Hyperirritability.
Symptoms manifest w/I 24/48 after birth, Exclude
hypocalcemia and hypoglycemia. 2-Mthadone withdrawl
resents at the 2nd to6th wek of life with seizure. 3-Cocaine:
asso w IUGR, intracranial hemorrhages, and premature labor
or abrupta placenta. Its not common. 4-Alcohol withdrawl:
presents with tremors, agitation, lethargy and seizures .Its
rare.
Urinary incontinence: Urinary incontinence is a common
problem in the elderly population which causes significant
functional and psychological morbidity. It frequently leads to a
decline in self-confidence and a diminished quality of life in
the elderly population.
It is important to distinguish between the three main types of
urinary incontinence to devise a proper treatment plan. A
detailed history and physical examination is the most
important step to differentiate between the various types of
urinary incontinence. A thorough history is also useful to
identify various reversible causes of urinary continence, such
as excessive fluid intake, urinary tract infection, atrophic
vaginitis, stool impaction, and the use of various drugs
causing or contributing to urinary incontinence.
The three major types of urinary incontinence are stress
incontinence, urge incontinence, and overflow urinary
incontinence. Stress incontinence is the most common cause
of urinary incontinence in younger females and occurs due to
inadequate urethral support from the fascia and muscles. An
increase in the intraabdominal pressure leads to urethral
sphincter opening, which causes urinary leakage in the

absence of bladder contraction. A detailed history, physical


examination, and clinical stress test can provide the diagnosis
of stress urinary incontinence in almost all the patients. In a
clinical stress test, the patient is asked to relax and give a
single vigorous cough. Leakage that occurs instantaneously
with coughing is virtually diagnostic of stress urinary
incontinence. Aggrevating factors are Obesity, pregnancy,
COPD and Smoking. Postvoid cystometry is normal. Tx include
Kegel excercise, esterogen in post menopasusal women.
Surgical tx is Burch and Sling procedures.
It is important to differentiate stress urinary incontinence
(urethral hypermobility) from urge incontinence (detrusor
overactivity). Pelvic floor exercises should always be the first
line of treatment in patients with stress urinary incontinence.
URGE Incontinence etruser instability, blader irritation form
neoplasm, and interestitial cyctitis result in UI, which causes
sudden and frequesnt loss of moderate to large amount of
urine. Often accomodated with Nocturia.
OVERFLOW: Diabetic Nephropathy causes OI. Characterized
by loss of small amount of urine from an over extended
bladder and a markedly increased residual volume. There is
hx of DM which is not controlled. CC are certain medications
(Ibuprofen), Diabetic nephropathy, MS and spinal cord injury.
NSAIDs have an inhibitory action on the detruser, so the first
step is to stop NSAID. Then cholinergic drug (Bethanechol)
should be added afterwards to improve detruser action .
Intermittent self catheterization can be used.
One of effects of epidural anesthesia is urinary retension due
to denervation of bladder. When bladder presure is >
sphingter pt urianates until balace is achieved again. This
incontinence is transient. PE may show distended blader.
Postvoidal vol is high. Tx is by Intermittant cathaterization
until control is regained. Oxybutyrin is used for Urge
incontinence. Urethroplexy is for stress incontinence.
Uterine prolapse: Uterine prolapse is typically seen in
multiparous, postmenopausal woman with a history of
multiple vaginal deliveries. The injury to the pelvic ligaments
and loss of estrogen weakens the endopelvic fascia. The
uterus and cervix descend down the vaginal canal towards the
vaginal orifice (introitus). Patients usually complain of a

sensation of pressure or heaviness in the pelvic area, which is


relieved by lying down and aggravated by prolonged standing
or exertion. Some patients may complain of low back pain,
dyspareunia, or a visible mass at the introitus. In chronic
cases, patients may have bleeding or discharge from
ulcerative, superficial epithelium. All symptomatic patients
(constant sensation of heaviness, pain, or bleeding) should
have surgical correction of the defect in the pelvic support.
The aim of surgical treatment is to completely relieve the
symptoms and prevent any future relapse.
Conservative treatment with the insertion of a pessary to hold
the pelvic organs in place should be only used in patients who
are poor candidates for surgical intervention. It does not
correct the underlying defect, and there is a higher chance of
relapse in active patients.

Alkaline phosphatase: Pregnancy is characterized by


increased alkaline phosphatase being secreted from the
placenta. Alkaline phosphatase is usually secreted by biliary
canalicular cells, placenta, bone, and intestinal mucosal cells.
Hence, raised alkaline phosphatase is a normal value in
growing children and pregnant women, provided there is no
associated symptoms or signs. Hence, reassuring the patient
and explaining this observation is adequate and no further
investigation is necessary.
Cervical cancer : Risk factors: Young age at first coitus (<20).
Young age at marriage and first pregnany. High parity,
multiple sex partners, smoking, and low socioeconomic
status.
If pap is dysplasia, perform colposcopy. If it shows
inflamatory Atypia then repeat after 4-6 weeks. If pt comes in
with spotting, and you see the cervix having a gross lesion
that bleeds by touching, dont even bother for PAP, go straight
to Punch biopsy to rule out cancer. **** Once pt had the
cancer check for cytology every year not every 2 year for

normal people.
Adenomyosis Is defined as presence of Endometrial glands in
the uterine muscle. MF in women above 49, , presents with
severe dysmenorrhea, and menorrhagia. The typica lexam
reveals enlarged sysmetrical uterus. If Adenomyosis is in one
side of uterus then enlargment is asymetrical.
DD: Myomatous Uterus , Leomyoma, Endometrial carcioma.
For women above 35, its mandatory to perform an
Endometrial curetage or even hysterectomyto rule out
endometrial cancer.Leomyomas, are difficult to ddx from
Adenomyosis, except that consistency of Uterus is softer in
Adenomyosis. Endometrial Carcinoma, occurs in women after
menopause . Endometritis manifest with fever, and enlarged
and tender uterus, asso with vaginal discharge . It usually
occurs after a septic abortion, and the mc oranism responsible
is Strep.
Fibroid uterus: Presents with Dysmenorrhea, heavy menses,
and enlarge uterus is almost dx of either Adenomyosis or FU.
Submucosal fibroids often imterefre with rmbryonal
implantation and infertility. Fibroids are the mc benign uterin
tmors in women and the mc indication for hysterectmy. Tey
are estrogen-dependent tumors, therefore they increase in
csize with OCP and pregnancy. and often regress after
menopause.
DD: Endometriosis which presents with Amenorrhea. Make
sure you can DDx the above conditions with Adenomyosis.
Granulosa Cell Tumor: SOLID tumors. Bimodal distribution. If
occur before puberty , Precociouspuberty is presented. It
produces excess estrogen and causes pubic hair, hpertrophy
of brest and hyperplasia of uterus. Usually removal of tumor
reverses the problem. If its in postmenopausal women it
causes bleedingand uterus shows myohyperplasia. DDX1
ysgerminoma, in young women and children, unilateral and
go under torsion. It doesnt produce any hormones.
DDX2:Sertoli-Leydig,produces androgen and
DEFEMINIZATION, followed by masculinization in childbearing
years. DDX3:Mature teratoma or Dermoid cysts, benign and
dont produce any hormones. DDX4;Serous cystadenomas, are
the mc CYSTIC ovarian neoplasm. 25% are malignant,half
cases are bilateral. They dont produce any hormones. Ovarian
mass and abdominla pain are presenting features.

Hirsutism Women produce androgens. DHEA-s and


Testosterone in adrenals and ovaries. DHEA-S is only in
adrenals by adrenal tumors. ACTH increase in pts w hirsutism
is seen with ectopic or pituitary dependant Cushings dis.
ACTH increases the production of cortisol as well as
angrogens from the adrenal glands, however, the adrenal
glands show diffuse hyperplasia rather than a discrete
adenoma.
Clomiphene Citrate: Is an antiestrogen acts by competitive
blocking of receptors of hypothalamous, inhibiting the
negative feedback that estrogenhas on GnRH and
consequently insreasing Lh & FSH and improving ovulation.
Along with hMg and hCG itsindicated for chronic ovulation. SE
are hot flashes, breast discomfort, spotting. DANAZOL is an
androgen derivative that has gonadotropin inhibitory effect .
Its indicated in Endometriosis, Fibroids and Fibrocystic breast
disease.
DES toxicity If given to pregnant women causesClear cell
ADENOCARCINOMA of vagina in their duaghters. In the old
days it was the best tx for threatened abortion. With erly dx
and tx survival is 80%.

Ovarian Cancer Screening for CA125 and vaginal US are good


eough for pt at risk of OC. Althout there is no evidence that
US of abdomen can help decrease mortality from OC.
Primary Dysmenorrhea Pt present with hx of lower abdominal
pain, that radiated to upper thighs and back. The pain is
colicky and starts a few hours prior to menses, lasting 3-4
days. It usually appears 6-12 months after menarche. Tx is
NSAID, so are OCPs. The pathology here is that menstural
fluid has higher levels of prostaglandins.
Vulvar Hypertrophic Dystrophy Thick and hyperkeratotic due
to long term scratching. Mostly seen in PM women.
Vulvar Papillomatosis or Condylomata acuminata. Caused by
HPV 6,11. Present as exophytic lesionswith araised
papilloatous or spiked surfacemay grow into a calliflowerlike

formation.

1
2
Next

Search Thread

Last

Login or Register to post messages

Similar forum topics


step by step - FROM STEP 1 LEARNING
TILL WORKING
High step 3 score and average step
1&2
step 1 topic worth revising for step 2
ck

user name

Related resources
USMLEWorld Step 2 CS Course
Kaplan USMLE Step 1 Web Prep
USMLEWorld Step 1 Qbank

Advertise | Support | Privacy | TOS | Premium | Contact

Login

Das könnte Ihnen auch gefallen